upper limb Flashcards

1
Q

A 45-year-old woman is being examined as a candidate
for cosmetic breast surgery. The surgeon notes
that both of her breasts sag considerably. Which structure
has most likely become stretched to result in this
condition?
⃣ A. Scarpa’s fascia
⃣ B. Pectoralis major muscle
⃣ C. Pectoralis minor muscle
⃣ D. Suspensory (Cooper’s) ligaments
⃣ E. Serratus anterior muscle

A

D. The suspensory ligaments of the breast, also
known as Cooper’s ligaments, are fi brous bands that
run from the dermis of the skin to the deep layer of
superfi cial fascia and are primary supports for the
breasts against gravity. Ptosis of the breast is usually
due to the stretching of these ligaments and can be
repaired with plastic surgery. Scarpa’s fascia is the
deep membranous layer of superfi cial fascia of the
anterior abdominal wall. The pectoralis major and
pectoralis minor are muscles that move the upper
limb and lie deep to the breast but do not provide any
direct support structure to the breast. The serratus
anterior muscle is involved in the movements of the
scapula.
GAS 131, 137-138; GA 62

How well did you know this?
1
Not at all
2
3
4
5
Perfectly
2
Q

A 27-year-old man was admitted to the emergency
department after an automobile collision in which he
suffered a fracture of the lateral border of the scapula.
Six weeks after the accident, physical examination reveals
weakness in medial rotation and adduction of the
humerus. Which nerve was most likely injured?
⃣ A. Lower subscapular
⃣ B. Axillary
⃣ C. Radial
⃣ D. Spinal accessory
⃣ E. Ulnar

A

A. Lower subscapular nerves arise from the cervical
spinal nerves 5 and 6. It innervates the subscapularis
and teres major muscles. The subscapularis and
teres major are both responsible for adducting and
medially rotating the arm. A lesion of this nerve
would result in weakness in these motions. The axillary
nerve also arises from cervical spinal nerves 5
and 6 and innervates the deltoid and teres minor
muscles. The deltoid muscle is large and covers the
entire surface of the shoulder, and contributes to arm
movement in any plane. The teres minor is a lateral
rotator and a member of the rotator cuff group of
muscles. The radial nerve arises from the posterior
cord of the brachial plexus. It is the largest branch,
and it innervates the triceps brachii and anconeus in
the arm. The spinal accessory nerve is cranial nerve
XI, and it innervates the trapezius muscle, which elevates
and depresses the scapula. The ulnar nerve
arises from the medial cord of the brachial plexus and
runs down the medial aspect of the arm. It innervates
muscles of the forearm and hand.
GAS 676-682; GA 366, 369

How well did you know this?
1
Not at all
2
3
4
5
Perfectly
3
Q

A 48-year-old female court stenographer is admitted
to the orthopedic clinic with symptoms of carpal
tunnel syndrome, with which she has suffered for almost
a year. Which muscles most typically become
weakened in this condition?
⃣ A. Dorsal interossei
⃣ B. Lumbricals III and IV
⃣ C. Thenar
⃣ D. Palmar interossei
⃣ E. Hypothenar

A

C. The thenar muscles (and lumbricals I and II) are
innervated by the median nerve, which runs through
the carpal tunnel. The carpal tunnel is formed anteriorly
by the fl exor retinaculum and posteriorly by the
carpal bones. Carpal tunnel syndrome is caused by a
compression of the median nerve, due to reduced
space in the carpal tunnel. The carpal tunnel contains
the tendons of fl exor pollicis longus, fl exor digitorum
profundus, and fl exor digitorum superfi cialis muscles.
The dorsal interossei, lumbricals III and IV, palmar interossei,
and hypothenar muscles are all innervated by
the ulnar nerve.
GAS 756-758, 764-765, 788; GA 8, 400

How well did you know this?
1
Not at all
2
3
4
5
Perfectly
4
Q

A 45-year-old male arrived at the emergency department
with injuries to his left elbow after he fell in
a bicycle race. Radiographic and MRI examinations
show a fracture of the medial epicondyle and a torn
ulnar nerve. Which of the following muscles would be
most likely to be paralyzed?
⃣ A. Flexor digitorum superfi cialis
⃣ B. Biceps brachii
⃣ C. Brachioradialis
⃣ D. Flexor carpi ulnaris
⃣ E. Supinator

A

D. Fracture of the medial epicondyle often causes
damage to the ulnar nerve due to its position in the
groove behind the epicondyle. The ulnar nerve innervates
one and a half muscles in the forearm—the
fl exor carpi ulnaris and the medial half of the fl exor
digitorum profundus. The nerve continues on to innervate
muscles in the hand. The fl exor digitorum
superfi cialis is innervated by the median nerve and
the biceps brachii by the musculocutaneous. The radial
nerve innervates both the brachioradialis and
supinator muscles.
GAS 724-730; GA 378, 390, 397-398

How well did you know this?
1
Not at all
2
3
4
5
Perfectly
5
Q

While walking to his classroom building, a fi rstyear
medical student slipped on the wet pavement and
fell against the curb, injuring his right arm. Radiographic
images showed a midshaft fracture of the humerus.
Which pair of structures was most likely injured
at the fracture site?
⃣ A. Median nerve and brachial artery
⃣ B. Axillary nerve and posterior humeral circum-
fl ex artery
⃣ C. Radial nerve and deep brachial artery
⃣ D. Suprascapular nerve and artery
⃣ E. Long thoracic nerve and lateral thoracic artery

A

C. A midshaft humeral fracture can result in injury
to the radial nerve and deep brachial artery because
they lie in the spiral groove located in the
midshaft. Injury to the median nerve and brachial
artery can be caused by a supracondylar fracture
that occurs by falling on an outstretched hand and
partially fl exed elbow. A fracture of the surgical neck
of the humerus can injure the axillary nerve and
posterior humeral circumfl ex artery. The suprascapular
artery and nerve can be injured in a shoulder
dislocation. The long thoracic nerve and lateral thoracic
artery may be damaged during a mastectomy
procedure.
GAS 713, 724; GA 361

How well did you know this?
1
Not at all
2
3
4
5
Perfectly
6
Q

An 18-year-old male is brought to the emergency
department after an injury while playing rugby. Imaging
reveals a transverse fracture of the humerus about
1 inch proximal to the epicondyles. Which nerve is
most frequently injured by the jagged edges of the broken
bone at this location?
⃣ A. Axillary
⃣ B. Median
⃣ C. Musculocutaneous
⃣ D. Radial
⃣ E. Ulnar

A

B. A supracondylar fracture often results in injury
to the median nerve. The course of the median nerve
is anterolateral, and at the elbow it lies medial to the
brachial artery on the brachialis muscle. The axillary
nerve passes posteriorly through the quadrangular
space, accompanied by the posterior circumfl ex humeral
artery, and winds around the surgical neck of
the humerus. Injury to the surgical neck may damage
the axillary nerve. The musculocutaneous nerve
pierces the coracobrachialis muscle and descends between
the biceps and brachialis muscle. It continues
into the forearm as the lateral antebrachial cutaneous
nerve. The ulnar nerve descends behind the medial
epicondyle in its groove and is easily injured and produces
“funny bone” symptoms.
GAS 724-731; GA 361

How well did you know this?
1
Not at all
2
3
4
5
Perfectly
7
Q

A 52-year-old band director suffered problems in
her right arm several days after strenuous fi eld exercises
for a major athletic tournament. Examination in
the orthopedic clinic reveals wrist drop and weakness
of grasp but normal extension of the elbow joint. There
is no loss of sensation in the affected limb. Which
nerve was most likely affected?
⃣ A. Ulnar
⃣ B. Anterior interosseous
⃣ C. Posterior interosseous
⃣ D. Median
⃣ E. Superfi cial radial

A

C. The radial nerve descends posteriorly between
the long and lateral heads of the triceps and passes
inferolaterally on the back of the humerus between
the medial and lateral heads of the triceps. It eventually
enters the anterior compartment and descends to
enter the cubital fossa, where it divides into superfi -
cial and deep branches. The deep branch of the radial
nerve winds laterally around the radius and runs
between the two heads of the supinator and continues
as the posterior interosseous nerve, innervating
extensor muscles of the forearm. Because this injury
does not result in loss of sensation over the skin of
the upper limb, it is likely that the superfi cial branch
of the radial nerve is not injured. If the radial nerve
were injured very proximally, the woman would not
have extension of her elbow. The branches of the
radial nerve to the triceps arise proximal to where the
nerve runs in the spiral groove. The anterior interosseous
nerve arises from the median nerve and supplies
the fl exor digitorum profundus, fl exor pollicis
longus, and pronator quadratus, none of which seem
to be injured in this example. Injury to the median
nerve causes a characteristic fl attening (atrophy) of
the thenar eminence.
GAS 750; GA 403

How well did you know this?
1
Not at all
2
3
4
5
Perfectly
8
Q

A 32-year-old woman is admitted to the emergency
department after an automobile collision. Radiographic
examination reveals multiple fractures of the
humerus. Flexion and supination of the forearm are
severely weakened. She also has loss of sensation on
the lateral surface of the forearm. Which of the following
nerves has most likely been injured?
⃣ A. Radial
⃣ B. Musculocutaneous
⃣ C. Median
⃣ D. Lateral cord of brachial plexus
⃣ E. Lateral cutaneous nerve of the forearm

A

B. The musculocutaneous nerve supplies the biceps
brachii and brachialis, which are the fl exors of the
forearm at the elbow. The musculocutaneous nerve
continues as the lateral antebrachial cutaneous nerve,
which supplies sensation to the lateral side of the forearm
(with the forearm in the anatomic position). The
biceps brachii is the most powerful supinator muscle.
Injury to this nerve would result in weakness of supination
and forearm fl exion and lateral forearm sensory
loss. Injury to the radial nerve would result in weakened
extension and a characteristic wrist drop. Injury to
the median nerve causes paralysis of fl exor digitorum
superfi cialis and other fl exors in the forearm and results
in a characteristic fl attening of the thenar eminence.
The lateral cord of the brachial plexus gives origin both
to the musculocutaneous and lateral pectoral nerves.
There is no indication of pectoral paralysis or weakness.
Injury to the lateral cord can result in weakened fl exion
and supination in the forearm, and weakened adduction
and medial rotation of the arm. The lateral cutaneous
nerve of the forearm is a branch of the musculocutaneous
nerve and does not supply any motor
innervation. Injury to the musculocutaneous nerve
alone is unusual but can follow penetrating injuries.
GAS 720; GA 361, 369-371, 376, 390

How well did you know this?
1
Not at all
2
3
4
5
Perfectly
9
Q

A 24-year-old medical student was bitten at the
base of her thumb by her dog. The wound became
infected and the infection spread into the radial
bursa. The tendon(s) of which muscle will most
likely be affected?
⃣ A. Flexor digitorum profundus
⃣ B. Flexor digitorum superfi cialis
⃣ C. Flexor pollicis longus
⃣ D. Flexor carpi radialis
⃣ E. Flexor pollicis brevis

A

C. Tenosynovitis can be due to an infection of the
synovial sheaths of the digits. Tenosynovitis in the
thumb may spread through the synovial sheath of
the fl exor pollicis longus tendon, also known as the
radial bursa. The tendons of the fl exor digitorum superfi
cialis and profundus muscles are enveloped in
the common synovial fl exor sheath, or ulnar bursa.
Neither the fl exor carpi radialis nor fl exor pollicis brevis
tendons are contained in synovial fl exor sheaths.
GAS 759-761; GA 397-399

How well did you know this?
1
Not at all
2
3
4
5
Perfectly
10
Q

Laboratory studies in the outpatient clinic on a
24-year-old female included assessment of circulating
blood chemistry. Which of the following arteries is
most likely at risk during venipuncture at the cubital
fossa?
⃣ A. Brachial
⃣ B. Common interosseous
⃣ C. Ulnar
⃣ D. Anterior interosseous
⃣ E. Radial

A

A. The three chief contents of the cubital fossa are
the biceps brachii tendon, brachial artery, and median
nerve (lateral to medial). The common and anterior
interosseous arteries arise distal to the cubital fossa; the
ulnar and radial arteries are the result of the bifurcation
of the brachial artery distal to the cubital fossa.
GAS 729; GA 361, 366-368, 380

How well did you know this?
1
Not at all
2
3
4
5
Perfectly
11
Q

A 22-year-old male is diagnosed with metastatic
malignant melanoma of the skin over the xiphoid process.
Which nodes receive most of the lymph from this
area and are therefore most likely to be involved in
metastasis of the tumor?
⃣ A. Deep inguinal
⃣ B. Vertical group of superfi cial inguinal
⃣ C. Horizontal group of superfi cial inguinal
⃣ D. Axillary
⃣ E. Deep and superfi cial inguinal

A

D. Lymph from the skin of the anterior chest
wall primarily drains to the axillary lymph nodes.
GAS 709; GA 12, 133

How well did you know this?
1
Not at all
2
3
4
5
Perfectly
12
Q

A 49-year-old female who had suffered a myocardial
infarction must undergo a bypass graft procedure
using the internal thoracic artery. Which vessels will
most likely continue to supply blood to the anterior
part of the upper intercostal spaces?
⃣ A. Musculophrenic
⃣ B. Superior epigastric
⃣ C. Posterior intercostal
⃣ D. Lateral thoracic
⃣ E. Thoracodorsal

A

C. The anterior intercostal arteries are 12 small
arteries, two in each of the upper six intercostal
spaces at the upper and lower borders. The upper
artery lying in each space anastomoses with the posterior
intercostal arteries, whereas the lower one usually
joins the collateral branch of the posterior intercostal
artery. The musculophrenic artery supplies the
pericardium, diaphragm, and muscles of the abdominal
wall. It anastomoses with the deep circumfl ex iliac
artery. The superior epigastric artery supplies the diaphragm,
peritoneum, and the anterior abdominal wall
and anastomoses with the inferior epigastric artery.
The lateral thoracic artery runs along the lateral border
of the pectoralis minor muscle and supplies the
pectoralis major, pectoralis minor, and serratus anterior.
The thoracodorsal artery accompanies the thoracodorsal
nerve in supplying the latissimus dorsi
muscle and lateral thoracic wall.
GAS 151-155; GA 68

How well did you know this?
1
Not at all
2
3
4
5
Perfectly
13
Q

A 22-year-old woman is admitted to the emergency
department in an unconscious state. The nurse
takes a radial pulse to determine the heart rate of the
patient. This pulse is felt lateral to which tendon?
⃣ A. Palmaris longus
⃣ B. Flexor pollicis longus
⃣ C. Flexor digitorum profundus
⃣ D. Flexor carpi radialis
⃣ E. Flexor digitorum superfi cialis

A

D. The location for palpation of the radial pulse is
lateral to the tendon of the fl exor carpi radialis, where
the radial artery can be compressed against the distal
radius. The radial pulse can also be felt in the anatomic
snuffbox between the tendons of the extensor pollicis
brevis and extensor pollicis longus muscles, where the
radial artery can be compressed against the scaphoid.
GAS 349-377; GA 374, 390, 397-398

How well did you know this?
1
Not at all
2
3
4
5
Perfectly
14
Q

A 45-year-old male is admitted to the hospital
after accidentally walking through a plate glass door
in a bar while intoxicated. Physical examination
shows multiple lacerations to the upper limb, with inability
to fl ex the distal interphalangeal joints of the
fourth and fi fth digits. Which of the following muscles
is most likely affected?
⃣ A. Flexor digitorum profundus
⃣ B. Flexor digitorum superfi cialis
⃣ C. Lumbricals
⃣ D. Flexor digitorum profundus and fl exor digitorum
superfi cialis
⃣ E. Interossei

A

A. The fl exor digitorum profundus is dually innervated
by the ulnar nerve to the medial phalanges
and the median nerve for the lateral phalanges. Because
of the superfi cial course of the ulnar nerve, it is
vulnerable to laceration. Such an injury would result
in an inability to fl ex the distal interphalangeal joints
of the fourth and fi fth digits. The fl exor digitorum
superfi cialis is innervated by the median nerve only,
and the course of this nerve runs too deep to be affected
by lacerations. The lumbricals function to fl ex
the MP joints and assist in extending the IP joints.
The interossei adduct and abduct the fi ngers.
GAS 736; GA 399, 402

How well did you know this?
1
Not at all
2
3
4
5
Perfectly
15
Q

A 24-year-old man is admitted with a wound to
the palm of his hand. He cannot touch the pads of his
fi ngers with his thumb but can grip a sheet of paper
between all fi ngers and has no loss of sensation on the
skin of his hand. Which of the following nerves has
most likely been injured?
⃣ A. Deep branch of ulnar
⃣ B. Anterior interosseous
⃣ C. Median
⃣ D. Recurrent branch of median
⃣ E. Deep branch of radial

A

D. The recurrent branch of the median nerve is
motor to the muscles of the thenar eminence, which
is an elevation caused by the abductor pollicis brevis,
fl exor pollicis brevis, and opponens pollicis. If the opponens
pollicis is paralyzed, one cannot oppose the
pad of the thumb to the pads of the other digits. The
recurrent branch does not have a cutaneous distribution.
Holding a piece of paper between the fi ngers is
a simple test of adduction of the fi ngers. These movements
are controlled by the deep branch of the ulnar
nerve, which is not injured in this patient.
GAS 770, 773; GA 414, 417

How well did you know this?
1
Not at all
2
3
4
5
Perfectly
16
Q

A 55-year-old male is examined in a neighborhood
clinic after receiving blunt trauma to his right
axilla in a fall. He has diffi culty elevating the right arm
above the level of his shoulder. Physical examination
shows the inferior angle of his right scapula protrudes
more than the lower part of the left scapula. The right
scapula protrudes far more when the patient pushes
against resistance. Which of the following neural structures
has most likely been injured?
⃣ A. The posterior cord of the brachial plexus
⃣ B. The long thoracic nerve
⃣ C. The upper trunk of the brachial plexus
⃣ D. The site of origin of the middle and lower
subscapular nerves
⃣ E. Spinal nerve roots C7, C8, and T1

A

B. “Winging” of the scapula occurs when the medial
border of the scapula lifts off the chest wall when
the patient pushes against resistance, such as a wall.
The serratus anterior muscle holds the medial border of
the scapula against the chest wall and is innervated by
the long thoracic nerve. The serratus anterior assists in
abduction of the arm above the horizontal plane.
GAS 690; GA 361

How well did you know this?
1
Not at all
2
3
4
5
Perfectly
17
Q

A mother tugs violently on her male child’s hand
to pull him out of the way of an oncoming car and the
child screams in pain. Thereafter, it becomes obvious
that the child cannot straighten his forearm at the elbow.
When the child is seen in the emergency department,
radiographic examination reveals a dislocation of the
head of the radius. Which of the following ligaments is
most likely directly associated with this injury?
⃣ A. Anular
⃣ B. Joint capsular
⃣ C. Interosseous
⃣ D. Radial collateral
⃣ E. Ulnar collateral

A

A. The anular ligament is a fi brous band that
encircles the head of the radius, forming a collar that
fuses with the radial collateral ligament and articular
capsule of the elbow. The anular ligament functions
to prevent displacement of the head of the radius
from its socket. The joint capsule functions to allow
free rotation of the joint and does not function in its
stabilization. The interosseous membrane is a fi brous
layer between the radius and ulna helping to hold
these two bones together. The radial collateral ligament
extends from the lateral epicondyle to the margins
of the radial notch of the ulnar and the anular
ligament of the radius. The ulnar collateral ligament
is triangular ligament and extends from the medial
epicondyle to the olecranon of the ulna.
GAS 724-729; GA 388

How well did you know this?
1
Not at all
2
3
4
5
Perfectly
18
Q

After a forceps delivery of a male infant, the baby
presents with his left upper limb adducted, internally
rotated, and fl exed at the wrist. The startle refl ex is absent
on the ipsilateral side. Which part of the brachial
plexus was most likely injured during this delivery?
⃣ A. Lateral cord
⃣ B. Medial cord
⃣ C. Roots of the lower trunk
⃣ D. Root of the middle trunk
⃣ E. Roots of the upper trunk

A

E. The injury being described is also known as
Erb-Duchenne paralysis or “waiter’s tip-hand.” This
usually results from an injury to the upper trunk of the
brachial plexus, presenting with loss of abduction,
fl exion, and lateral rotation of the arm. The superior
trunk of the brachial plexus consists of spinal nerve
roots C5-6.
GAS 700-709; GA 361, 369-371

How well did you know this?
1
Not at all
2
3
4
5
Perfectly
19
Q

A 35-year-old patient has a small but painful tumor
under the nail of the little fi nger. Which of the
following nerves would have to be anesthetized for a
painless removal of the tumor?
⃣ A. Superfi cial radial
⃣ B. Common palmar digital of median
⃣ C. Common palmar digital of ulnar
⃣ D. Deep radial
⃣ E. Recurrent branch of median

A

C. The common palmar digital branch comes off
the superfi cial branch of the ulnar nerve and supplies
the skin of the little fi nger and the medial side of the
ring fi nger. The superfi cial branch of the radial nerve
provides cutaneous innervation to the radial (lateral)
dorsum of the hand and the radial two and a half
digits over the proximal phalanx. The common palmar
digital branch of the median nerve innervates
most of the lateral aspect of the palmar hand and the
dorsal aspect of the second and third fi nger as well as
the lateral part of the fourth digit. The deep radial
nerve supplies the extensor carpi radialis brevis and
supinator muscles and continues as the posterior interosseous
nerve. The recurrent branch of the median
nerve supplies the abductor pollicis brevis, fl exor pollicis
brevis, and opponens pollicis muscles.
GAS 744; GA 414

How well did you know this?
1
Not at all
2
3
4
5
Perfectly
20
Q

A 25-year-old male athlete is admitted to the
emergency department after a bad landing in the pole
vault. Radiographic examination of his hand reveals a
fractured carpal bone in the fl oor of the anatomic
snuffbox ( Fig. 6-1 ). Which bone has most likely been
fractured?
⃣ A. Triquetral
⃣ B. Scaphoid
⃣ C. Capitate
⃣ D. Hamate
⃣ E. Trapezoid

A

B. The anatomic snuffbox is formed by the tendons
of the extensor pollicis brevis, the abductor pollicis
longus, and the extensor pollicis longus. The fl oor
is formed by the scaphoid bone, and it is here that one
can palpate for a possible fractured scaphoid.
GAS 752-754; GA 392, 394, 422

How well did you know this?
1
Not at all
2
3
4
5
Perfectly
21
Q

A 36-year-old man is brought to the emergency
department because of a deep knife wound on the medial
side of his distal forearm. He is unable to hold a
piece of paper between his fi ngers and has sensory loss
on the medial side of his hand and little fi nger. Which
nerve is most likely injured?
⃣ A. Axillary
⃣ B. Median
⃣ C. Musculocutaneous
⃣ D. Radial
⃣ E. Ulnar

A

E. The ulnar nerve innervates the palmar interossei,
which adduct the fi ngers. This is the movement
that would maintain the paper between the fi ngers.
The axillary nerve does not innervate muscles of the
hand. The median nerve supplies the fi rst and second
lumbricals, the opponens pollicis, abductor pollicis
brevis, and the fl exor pollicis brevis. None of these
muscles would affect the ability to hold a piece of paper
between the fi ngers. The musculocutaneous and
radial nerves do not supply muscles of the hand.
GAS 661, 706, 720; GA 369-371, 390, 400, 417

How well did you know this?
1
Not at all
2
3
4
5
Perfectly
22
Q

A 19-year-old man is brought to the emergency
department after dislocating his shoulder while playing
soccer. Following reduction of the dislocation, he has
pain over the dorsal region of the shoulder and cannot
abduct the arm normally. An MRI of the shoulder
shows a torn muscle. Which of the following muscles
is most likely to have been damaged by this injury?
⃣ A. Coracobrachialis
⃣ B. Long head of the triceps
⃣ C. Pectoralis minor
⃣ D. Supraspinatus
⃣ E. Teres major

A

D. The supraspinatus is one of the rotator cuff
muscles. Its tendon is relatively avascular and is often
injured when the shoulder is dislocated. This muscle
initiates abduction of the arm, and damage would
impair this movement. The coracobrachialis muscle,
which runs from the coracoid process to the humerus,
functions in adduction and fl exion of the arm. The
triceps’ main function is to extend the elbow, and
damage to its long head would not affect abduction.
The pectoralis minor functions as an accessory respiratory
muscle and to stabilize the scapula and is not
involved in abduction. The teres major functions to
adduct and medially rotate the arm.
GAS 678-680; GA 38-39, 361, 364

How well did you know this?
1
Not at all
2
3
4
5
Perfectly
23
Q

A 47-year-old female tennis professional is informed
by her physician that she has a rotator cuff injury
that will require surgery. Her physician explains
that over the years of play a shoulder ligament has
gradually caused severe damage to the underlying
muscle. To which of the following ligaments is the physician
most likely referring?
⃣ A. Acromioclavicular ligament
⃣ B. Coracohumeral ligament
⃣ C. Transverse scapular ligament
⃣ D. Glenohumeral ligament
⃣ E. Coracoacromial ligament

A

E. The coracoacromial ligament contributes to
the coracoacromial arch, preventing superior displacement
of the head of the humerus. Because this ligament
is very strong, it will rarely be damaged; instead,
the ligament can cause infl ammation or erosion of the
tendon of the supraspinatus muscle as the tendon
passes back and forth under the ligament. The acromioclavicular
ligament, connecting the acromion with
the lateral end of the clavicle, is not in contact with the
supraspinatus tendon. The coracohumeral ligament is
located too far anteriorly to impinge upon the supraspinatus
tendon. The glenohumeral ligament is located
deep to the rotator cuff muscles and would not contribute
to injury of the supraspinatus muscle. The transverse
scapular ligament crosses the scapular notch and
is not in contact with the supraspinatus tendon.
GAS 665; GA 354, 356

How well did you know this?
1
Not at all
2
3
4
5
Perfectly
24
Q

A 69-year-old man has numbness in the middle
three digits of his right hand and fi nds it diffi cult to
grasp objects with that hand. He states that he retired
9 years earlier, after working as a carpenter for
50 years. He has atrophy of the thenar eminence ( Fig.
6-2 ). Which of the following conditions is the most
likely cause of the problems in his hand?
⃣ A. Compression of the median nerve in the carpal
tunnel
⃣ B. Formation of the osteophytes that compress
the ulnar nerve at the medial epicondyle
⃣ C. Hypertrophy of the triceps muscle compressing
the brachial plexus
⃣ D. Osteoarthritis of the cervical spine
⃣ E. Repeated trauma to the ulnar nerve

A

A. The median nerve supplies sensory innervation
to the thumb, index, and middle fi ngers and also
to the lateral half of the ring fi nger. The median nerve
also provides motor innervation to muscles of the thenar
eminence. Compression of the median nerve in the
carpal tunnel explains these defi cits in conjunction
with normal functioning of the fl exor compartment of
the forearm because these muscles are innervated by
the median nerve proximal to the carpal tunnel. The
ulnar nerve is not implicated in these symptoms. It
does not provide sensation to digits 1 to 3. Compression
of the brachial plexus could not be attributed to
pressure from the triceps because this muscle is located
distal to the plexus. In addition, brachial plexus
symptoms would include other upper limb defi cits,
rather than the focal symptoms described in this case.
Osteoarthritis of the cervical spine would also lead to
increasing complexity of symptoms.
GAS 764, 788; GA 406

How well did you know this?
1
Not at all
2
3
4
5
Perfectly
25
A 13-year-old boy is brought to the emergency department after losing control during a motorbike race in which he was run over by several of the other racers. Physical examination reveals several cuts and bruises. He is unable to extend the left wrist, fi ngers, and thumb, although he can extend the elbow. Sensation is lost in the lateral half of the dorsum of the left hand. Which of the following nerves has most likely been injured to result in these signs, and in what part of the arm is the injury located? ⃣ A. Median nerve, anterior wrist ⃣ B. Median nerve, arm ⃣ C. Radial nerve, midhumerus ⃣ D. Ulnar nerve, midlateral forearm ⃣ E. Ulnar nerve, midpalmar region
C. The radial nerve innervates the extensor compartments of the arm and the forearm. It supplies the triceps brachii proximal to the spiral groove, so elbow extension is intact here. It also provides sensory innervation to much of the posterior arm and forearm as well as the dorsal thumb, index, and middle fi ngers up to the level of the fi ngernails. Symptoms are described only in the distal limb due to the midhumeral location of the lesion. The median nerve innervates fl exors of the forearm and thenar muscles and provides sensory innervation to the lateral palmar hand. The ulnar nerve supplies only the fl exor carpi ulnaris and the medial half of the fl exor digitorum profundus in the forearm. Additionally, its sensory distribution is to both the palmar and dorsal aspects of the medial hand. It does not supply extensor muscles. GAS 713, 724; GA 361
26
A 17-year-old male has weakness of elbow fl exion and supination of the left hand after sustaining a knife wound in that arm in a street fi ght. Examination in the emergency department indicates that a nerve has been severed. Which of the following conditions will also most likely be seen during physical examination? ⃣ A. Inability to adduct and abduct his fi ngers ⃣ B. Inability to fl ex his fi ngers ⃣ C. Inability to fl ex his thumb ⃣ D. Sensory loss over the lateral surface of his forearm ⃣ E. Sensory loss over the medial surface of his forearm
D. The musculocutaneous nerve innervates the brachialis and biceps brachii muscles, which are the main fl exors at the elbow. The biceps inserts on the radius and is an important supinator. Because the musculocutaneous nerve is damaged in this case, it leads to loss of sensory perception to the lateral forearm, which is supplied by the distal portion of the musculocutaneous nerve (known as the lateral antebrachial cutaneous nerve). Adduction and abduction of the fi ngers are mediated by the ulnar nerve and would not be affected in this instance. The fl exor pollicis brevis fl exes the thumb and is mainly innervated by the recurrent branch of the median nerve. Flexion of the fi ngers is performed by the long fl exors of the fi ngers and lumbrical muscles, innervated by the median and ulnar nerves. Sensory innervation of the medial forearm is provided by the medial antebrachial cutaneous nerve, a branch of the medial cord of the brachial plexus. GAS 744; GA 424-425
27
Following several days of 12-hour daily rehearsals of the symphony orchestra for a performance of a Wagnerian opera, the 52-year-old male conductor experienced such excruciating pain in the posterior aspect of his right forearm that he could no longer direct the musicians. When the maestro’s forearm was palpated 2 cm distal to, and posteromedial to, the lateral epicondyle, the resulting excruciating pain caused the conductor to weep. Injections of steroids and rest were recommended to ease the pain. Which of the following injuries is most likely? ⃣ A. Compression of the ulnar nerve by the fl exor carpi ulnaris ⃣ B. Compression of the median nerve by the pronator teres ⃣ C. Compression of the median nerve by the fl exor digitorum superfi cialis ⃣ D. Compression of the superfi cial radial nerve by the brachioradialis ⃣ E. Compression of the deep radial nerve by the supinator
E. The deep radial nerve courses between the two heads of the supinator and is located just medial and distal to the lateral epicondyle. It can be irritated by hypertrophy of the supinator, which compresses the nerve, causing pain and weakness. The ulnar nerve courses laterally behind the medial epicondyle and continues anterior to the fl exor carpi ulnaris. The median nerve passes into the forearm fl exor compartment; the superfi cial radial nerve courses down the lateral aspect of the posterior forearm and would not cause pain due to pressure applied to the posterior forearm. GAS 747; GA 390
28
A 54-year-old female marathon runner presents with pain in her right wrist that resulted when she fell with force on her outstretched hand. Radiographic studies indicate an anterior dislocation of a carpal bone ( Fig. 6-3 ). Which of the following bones is most likely dislocated? ⃣ A. Capitate ⃣ B. Lunate ⃣ C. Scaphoid ⃣ D. Trapezoid ⃣ E. Triquetrum
B. The lunate is the most commonly dislocated carpal bone because of its shape and relatively weak ligaments anteriorly. Dislocations of the scaphoid and triquetrum are relatively rare. The trapezoid and capitate bones are located in the distal row of the carpal bones. GAS 752-754; GA 392, 394, 422
29
A 45-year-old man is admitted to the hospital after a car crash. Radiographic examination reveals mild disk herniations of C7, C8, and T1. The patient presents with a sensory defi cit of the C8 and T1 spinal nerve dermatomes. The dorsal root ganglia of C8 and T1 would contain cell bodies of sensory fi bers carried by which of the following nerves? ⃣ A. Medial antebrachial cutaneous nerve ⃣ B. Long thoracic nerve ⃣ C. Lateral antebrachial cutaneous nerve ⃣ D. Deep branch of ulnar nerve ⃣ E. Anterior interosseous nerve
A. The medial antebrachial cutaneous nerve carries sensory fi bers derived from the C8 and T1 levels. The lateral antebrachial cutaneous nerve is the distal continuation of the musculocutaneous nerve, carrying fi bers from the C5, C6, and C7 levels. The deep branch of the ulnar nerve and the anterior interosseous nerves carry predominantly motor fi bers. The sensory fi bers coursing in the radial nerve are derived from the C5 to C8 levels. GAS 700-709; GA 361, 369-371
30
A 23-year-old female maid was making a bed in a hotel bedroom. As she straightened the sheet by running her right hand over the surface with her fi ngers extended, she caught the end of the index fi nger in a fold. She experienced a sudden, severe pain over the base of the terminal phalanx. Several hours later when the pain had diminished, she noted that the end of her right index fi nger was swollen and she could not completely extend the terminal interphalangeal joint. Which one of the following structures within the digit was most likely injured? ⃣ A. The proper palmar digital branch of the median nerve ⃣ B. The vinculum longa ⃣ C. The insertion of the tendon of the extensor digitorum onto the base of the distal phalanx ⃣ D. The insertion of the fl exor digitorum profundus tendon ⃣ E. The insertion of the fl exor digitorum superfi - cialis tendon
C. The contraction of the extensor mechanism produces extension of the distal interphalangeal joint. When it is torn from the distal phalanx, the digit is pulled into fl exion by the fl exor digitorum profundus. The proper palmar digital branches of the median nerve supply lumbrical muscles and carry sensation from their respective digits. Vincula longa are slender, bandlike connections from the deep fl exor tendons to the phalanx that can carry blood supply to the tendons. The insertions of the fl exor digitorum superfi - cialis and profundus are on the fl exor surface of the middle and distal phalanges, respectively, and act to fl ex the interphalangeal joints. GAS 745-747; GA 378, 401
31
A 45-year-old patient had fallen upon his outstretched hand, resulting in a Smith fracture of the distal end of the radius. The fractured bone displaced a carpal bone in the palmar direction, resulting in nerve compression within the carpal tunnel. Which of the following carpal bones will most likely be dislocated? ⃣ A. Scaphoid ⃣ B. Trapezium ⃣ C. Capitate ⃣ D. Hamate ⃣ E. Lunate
E. In a Smith fracture, the distal fragment of the radius deviates palmarward, often displacing the lunate bone. The other listed bones are unlikely to be displaced in a palmar direction by a Smith fracture. GAS 752-754; GA 392, 394, 422
32
A 15-year-old girl was brought to the emergency department with a tear of the tendons in the fi rst dorsal compartment of the wrist from a severe bite by a pit bull dog. The injured tendons in this compartment would include which of the following muscles? ⃣ A. Extensor carpi radialis longus and brevis ⃣ B. Abductor pollicis longus and extensor pollicis brevis ⃣ C. Extensor digitorum ⃣ D. Extensor indicis proprius ⃣ E. Extensor carpi ulnaris
B. The abductor pollicis longus and extensor pollicis brevis are the occupants of the fi rst dorsal compartment of the wrist. The extensor carpi radialis longus and brevis are in the second compartment. The extensor digitorum is in the third compartment, as is the extensor indicis proprius. The extensor carpi ulnaris is located in the sixth dorsal compartment. GAS 748-749; GA 401-402, 412-413
33
As she fell from the uneven parallel bars, the 17- year-old female gymnast grasped the lower bar briefl y with one hand but then fell painfully to the fl oor. An MRI examination reveals an injury to the medial cord of the brachial plexus. Which of the following spinal nerve levels would most likely be affected? ⃣ A. C5, C6 ⃣ B. C6, C7 ⃣ C. C7, C8 ⃣ D. C7, C8, T1 ⃣ E. C8, T1
E. The medial cord has been injured by traction on the lower trunk of the brachial plexus. The medial cord is the continuation of the inferior (lower) trunk of the brachial plexus, which is formed by C8 and T1. C5 and C6 are typically associated with the superior (upper) trunk level and thus the lateral cord. C7 forms the middle trunk. An injury to the posterior cord would usually involve the C7 spinal nerve. This is a typical Klumpke paralysis. GAS 700-709; GA 361, 369-371
34
A 21-year-old female softball pitcher is examined in the emergency department after she was struck in the arm by a line drive (a ball hit very hard and low). Radiographic and MRI studies show soft tissue injury to the region of the spiral groove, with trauma to the radial nerve. Which of the following muscles would be intact after this injury? ⃣ A. Flexor carpi ulnaris ⃣ B. Extensor indicis ⃣ C. Brachioradialis ⃣ D. Extensor carpi radialis ⃣ E. Supinator
A. The fl exor carpi ulnaris muscle is not innervated by the radial nerve but rather by the ulnar UPPER LIMB 195 nerve. The brachioradialis, extensor carpi radialis, and supinator muscles are all innervated by the radial nerve distal to the spiral groove. GAS 737-739; GA 378, 390, 397, 398
35
Examination of a 21-year-old female athlete with an injury of the radial nerve in the spiral groove would typically demonstrate which of the following physical signs? ⃣ A. Weakness of thumb abduction and thumb extension ⃣ B. Weakness of thumb opposition ⃣ C. Inability to extend the elbow ⃣ D. Paralysis of pronation of the hand ⃣ E. Paralysis of abduction and adduction of the arm
A. Injury to the radial nerve in the spiral groove will paralyze the abductor pollicis longus and both extensors of the thumb. This injury will also lead to wrist drop (inability to extend the wrist). Weakness of grip would also occur, although this is not mentioned in the question. If the wrist is fl exed, fi nger fl exion and grip strength are weakened because the long fl exor tendons are not under tension. Note how much your strength of grip is increased when your wrist is extended versus when it is fl exed. GAS 661, 709, 722-724; GA 366, 371
36
The 58-year-old convenience store operator had received a superfi cial bullet wound to the soft tissues on the medial side of the elbow in an attempted robbery. A major nerve was repaired at the site where it passed behind the medial epicondyle. Bleeding was stopped from an artery that accompanied the nerve in its path toward the epicondyle. Vascular repair was performed on this small artery because of its important role in supplying blood to the nerve. Which of the following arteries was most likely repaired? ⃣ A. The profunda brachii ⃣ B. The radial collateral artery ⃣ C. The superior ulnar collateral artery ⃣ D. The inferior ulnar collateral artery ⃣ E. The anterior ulnar recurrent artery
C. The superior ulnar collateral branch of the brachial artery accompanies the ulnar nerve in its path posterior to the medial epicondyle and is important in the blood supply of the nerve. The profunda brachii passes down the arm with the radial nerve. The radial collateral artery arises from the profunda brachii and anastomoses with the radial recurrent branch of the radial artery proximal to the elbow laterally. The inferior ulnar collateral artery arises from the brachial artery and accompanies the median nerve into the forearm. The anterior ulnar recurrent artery arises from the ulnar artery and anastomoses with the inferior ulnar collateral anterior to the elbow. GAS 743, 767-769; GA 368
37
A 60-year-old male butcher accidentally slashed his wrist with his butcher knife, partially dividing the ulnar nerve. Which of the following actions would most likely be lost as a result of this injury? ⃣ A. Flexion of the proximal interphalangeal joint of the fi fth digit (little fi nger) ⃣ B. Extension of the thumb ⃣ C. Adduction of the fi fth digit ⃣ D. Abduction of the thumb ⃣ E. Opposition of the thumb
C. Adduction of the fi fth digit is produced by contraction of the third palmar interosseous muscle. All of the interossei are innervated by the deep branch of the ulnar nerve. Flexion of the proximal interphalangeal joint is a function of the fl exor digitorum superfi cialis, supplied by the median nerve. Opposition of the thumb is a function of the opponens pollicis, supplied by the recurrent branch of the median nerve. GAS 729, 772-774; GA 414
38
A 23-year-old male medical student fell asleep in his chair with Netter’s Atlas wedged into his axilla. When he awoke in the morning, he was unable to extend the forearm, wrist, or fi ngers. Movements of the ipsilateral shoulder joint appear to be normal. Which of UPPER LIMB 175 the following nerves was most likely compressed, producing the symptoms described? ⃣ A. Lateral cord of the brachial plexus ⃣ B. Medial cord of the brachial plexus ⃣ C. Radial nerve ⃣ D. Median nerve ⃣ E. Lateral and medial pectoral nerves
C. The radial nerve is the most likely nerve compressed to cause these symptoms. This type of nerve palsy is often called “Saturday night palsy.” One reason for this nickname is that people would supposedly fall asleep after being intoxicated on a Saturday night with their arm over the back of a chair, thereby compressing the nerve in the spiral groove. The radial nerve innervates all of the extensors of the elbow, wrist, and fi ngers. Paralysis of the lateral cord of the brachial plexus would result in loss of the musculocutaneous nerve and the pectoral nerves, which do not mediate extension of the forearm or hand. The medial cord of the brachial plexus branches into the median nerve and ulnar nerve. Neither of these nerves innervates muscles that control extension. The median nerve innervates fl exors of the forearm and the thenar muscles. The lateral and median pectoral nerves do not extend into the arm and innervate the pectoralis major and minor muscles. GAS 789-790; GA 366, 371
39
The fact that the kidneys of a 32-year-old female patient were failing required that she be placed on dialysis. However, the search in her upper limb for a suitable vein was unexpectedly diffi cult. The major vein on the lateral side of the arm was too small; others were too delicate. Finally, a vein was found on the medial side of the arm that passed through the superfi cial and deep fascia to join veins beside the brachial artery. Which of the following veins was this? ⃣ A. Basilic ⃣ B. Lateral cubital ⃣ C. Cephalic ⃣ D. Medial cubital ⃣ E. Medial antebrachial
A. The basilic vein can be used for dialysis, especially when the cephalic vein is judged to be too small, as in this case. The basilic vein can be elevated from its position as it passes through the fascia on the medial side of the arm. The cephalic vein passes more laterally up the limb. The lateral cubital vein is a tributary to the cephalic vein, and the medial cubital vein joins the basilic vein—both rather superfi cial in position. The medial antebrachial vein courses up the midline of the forearm (antebrachium) ventrally. GAS 663, 697, 720, 770; GA 350, 360, 377
40
A 29-year-old female had sustained a deep laceration in the proximal part of the forearm. After the wound is closed, the following functional defi cits are observed by the neurologist on the service: The fi rst three digits are in a position of extension and cannot be fl exed. Digits 4 and 5 are partially fl exed at the metacarpophalangeal joints and noticeably more fl exed at the distal interphalangeal joints. Sensation is absent in the lateral side of the palm and the palmar surfaces of digits 1 to 3 and half of the fourth digit. Which of the following nerves has (have) most likely been injured? ⃣ A. Median nerve ⃣ B. Ulnar and median nerve ⃣ C. Ulnar nerve ⃣ D. Radial and ulnar nerve ⃣ E. Radial nerve
A. The patient exhibits the classic “benediction attitude” of the thumb and fi ngers from injury to the median nerve proximally in the forearm. The thumb is somewhat extended (radial supplied abductor and extensors unopposed); digits 2 and 3 are extended (by intact interossei); digits 4 and 5 are partially fl exed (by their intact fl exor digitorum profundus). A lesion of the median nerve would result in weakened fl exion of the PIP joints of all digits (fl exor digitorum superfi cialis), loss of fl exion of the interphalangeal joint of the thumb, the DIP joints of digits 2 and 3 (fl exor digitorum profundus), and weakened fl exion of the metacarpophalangeal joints of the second and third digits (fi rst and second lumbricals). A lesion of both the ulnar and median nerves would cause weakness or paralysis of fl exion of all of the digits. A lesion of the ulnar nerve would mostly cause weakness in fl exion of the DIP of the fourth and fi fth digits and would affect all of the interosseous muscles and the lumbricals of the third and fourth digits. A lesion of the radial nerve would cause weakness in extension of the wrist, thumb, and metacarpophalangeal joints. GAS 724, 789-790; GA 371, 376
41
A 35-year-old male wrestler is admitted to the emergency department with excruciating pain in his right shoulder and proximal arm. During physical examination the patient clutches the arm at the elbow with his opposite hand and is unable to move the injured limb. Radiographic studies show that the patient has a dislocation of the humerus at the glenohumeral joint. Which of the following conditions is the most likely? ⃣ A. The head of the humerus is displaced anteriorly. ⃣ B. The head of the humerus is displaced posteriorly. ⃣ C. The head of the humerus is displaced inferiorly. ⃣ D. The head of the humerus is displaced superiorly. ⃣ E. The head of the humerus is displaced medially.
C. The head of the humerus is displaced inferiorly because in that location it is not supported by rotator cuff muscle tendons or the coracoacromial arch. It is also pulled anteriorly beneath the coracoid process by pectoralis and subscapularis muscles. It would not be displaced posteriorly because it is supported by the teres minor and infraspinatus muscle tendons. It would not be displaced superiorly because the acromioclavicular ligament and supraspinatus reinforce in that direction. A medial dislocation is blocked by the subscapularis tendon. GAS 667; GA 355
42
The 35-year-old female patient has a hard nodule about 1 cm in diameter slightly above and lateral to her right areola. A specifi c dye is injected into the tissue around the tumor, and an incision is made to expose the lymphatic vessels draining the area, for the lymphatic vessels take up the dye—which is visible to the eye. The vessels can then be traced to surgically expose the lymph nodes receiving the lymph from the tumor. Which of the following nodes will most likely fi rst encounter the lymph from the tumor? ⃣ A. Anterior axillary (pectoral) nodes ⃣ B. Rotter interpectoral nodes ⃣ C. Parasternal nodes along the internal thoracic artery and vein ⃣ D. Central axillary nodes ⃣ E. Apical or infraclavicular nodes
A. The anterior axillary (or anterior pectoral) nodes are the fi rst lymph nodes to receive most of the lymph from the breast parenchyma, areola, and nipple. From there, lymph fl ows through central axillary, apical, and supraclavicular nodes in sequence. Rotter’s nodes lie between the pectoral muscles and are, unfortunately, an alternate route in some patients, speeding the rate of metastasis. The parasternal nodes receive lymph from the medial part of the breast and lie along the internal thoracic artery and vein. GAS 709; GA 63
43
During a fi ght in a tavern, a 45-year-old male construction worker received a shallow stab wound from a broken beer bottle at a point near the middle of the left posterior triangle of his neck. Upon physical examination it is observed that the left shoulder is drooping lower than the right shoulder, and the superior angle of the scapula juts out slightly. Strength in turning the head to the right or left appears to be symmetric. Which of the following nerves is most likely injured? ⃣ A. Suprascapular nerve in the supraspinous fossa ⃣ B. The terminal segment of the dorsal scapular nerve ⃣ C. The upper trunk of the brachial plexus ⃣ D. The spinal accessory nerve in the posterior cervical triangle ⃣ E. The thoracodorsal nerve in the axilla
D. The left spinal accessory nerve has been injured distal to the sternocleidomastoid muscle, resulting in paralysis of the trapezius, allowing the shoulder to droop and the superior angle to push out posteriorly. The sternocleidomastoid muscles are intact, as demonstrated by symmetry in strength in turning the head to the right and left. There is no indication of paralysis of the lateral rotators of the shoulder or elbow fl exors (suprascapular nerve or upper trunk). Thoracodorsal nerve injury would result in paralysis of the latissimus dorsi, an extensor, and medial rotator of the humerus. GAS 667, 973-974; GA 370
44
A 44-year-old woman is diagnosed with radial nerve palsy. When muscle function is examined at the metacarpophalangeal (MCP), proximal interphalangeal (PIP), and distal interphalangeal (DIP) joints, what fi ndings are most likely to be present? ⃣ A. Inability to abduct the digits at the MCP joint ⃣ B. Inability to adduct the digits at the MCP joint ⃣ C. Inability to extend the MCP joints only ⃣ D. Inability to extend the MCP, PIP, and DIP joints ⃣ E. Inability to extend the PIP and DIP joints
C. Inability to extend MCP joints. The tendons of the extensor digitorum and extensor digiti minimi, innervated by the radial nerve, are responsible for extension of the MCP, and to a much lesser degree, the PIP and DIP joints. Abduction and adduction of the MCP joints are functions of the interossei, all of which are innervated by the deep ulnar nerve. Extension of the PIP and DIP joints is performed by the lumbricals and interossei. The fi rst two lumbricals are supplied by the median nerve; the other lumbricals and the interossei, by the deep branch of the ulnar nerve. GAS 655; GA 395
45
A 27-year-old male painter is admitted to the hospital after falling from a ladder. Physical examination reveals that the patient is unable to abduct his arm more than 15 ° and cannot rotate the arm laterally. A radiographic examination reveals an oblique fracture of the humerus. He has associated sensory loss over the shoulder area. Which of the following injuries will most likely correspond to the symptoms of the physical examination? ⃣ A. Fracture of the medial epicondyle ⃣ B. Fracture of the glenoid fossa ⃣ C. Fracture of the surgical neck of the humerus ⃣ D. Fracture of the anatomic neck of the humerus ⃣ E. Fracture of the middle third of the humerus
C. Fracture of the surgical neck of the humerus often injures the axillary nerve, which innervates the deltoid and teres minor muscles. Abduction of the humerus between 15 ° and the horizontal is performed by the deltoid muscle. Lateral rotation of the humerus is mainly performed by the deltoid muscle, teres minor, and the infraspinatus. The deltoid and teres minor are both lost in this case. Fracture of the glenoid fossa would lead to drooping of the shoulder. Fracture of the anatomic neck of the humerus will similarly lead to a drooping of the shoulder but would not necessarily affect abduction of the humerus. It is also quite unusual. Fracture of the middle third of the humerus would most likely injure the radial nerve. The ulnar nerve would be potentially compromised in a fracture of the medial epicondyle of the humerus. GAS 667, 668; GA 355
46
A 47-year-old female patient’s right breast exhibited characteristics of peau d’orange; that is, the skin resembled orange peel. This condition is primarily a result of which of the following? ⃣ A. Shortening of the suspensory ligaments by cancer in the axillary tail ⃣ B. Blockage of cutaneous lymphatic vessels ⃣ C. Contraction of the retinacula cutis of the areola and nipple ⃣ D. Invasion of the pectoralis major by the cancer ⃣ E. Ipsilateral (same side) inversion of the periareolar skin from ductular cancer
B. When cutaneous lymphatics of the breast are blocked by cancer, the skin becomes edematous, except where hair follicles cause small indentations of the skin, giving an overall resemblance to orange peel. Shortening of the suspensory ligaments or retinacula cutis leads to pitting of the overlying skin, pitting that is intensifi ed if the patient raises her arm above her head. Invasion of the pectoralis major by cancer can result in fi xation of the breast, seen upon elevation of the ipsilateral limb. Inversion of areolar skin with involvement of the ducts would also be due to involvement of the retinacula cutis. GAS 709; GA 381
47
A 29-year-old female is examined in the emergency department after falling from her balcony. Radiographic examination reveals that she has suffered a broken clavicle, with associated internal bleeding. Which of the following vessels is most likely to be injured in clavicular fractures? ⃣ A. Subclavian artery ⃣ B. Cephalic vein ⃣ C. Lateral thoracic artery ⃣ D. Subclavian vein ⃣ E. Internal thoracic artery
D. The subclavian vein traverses between the clavicle and fi rst rib and is the most superfi cial structure to be damaged following a fracture of the clavicle. The subclavian artery runs deep to the subclavian vein, and though it is in the appropriate location, it would likely not be damaged because of its deep anatomic position. The cephalic vein is a tributary to the axillary vein after ascending on the lateral side of the arm. Its location within the body is too superfi cial and lateral to the site of injury. The lateral thoracic artery is a branch from the axillary artery that runs lateral to the pectoralis minor. It courses inferior and medial from its point of origin from the axillary artery, and it does not maintain a position near the clavicle during its descent. The internal thoracic artery arises from the fi rst part of the subclavian artery before descending deep to the costal cartilages. Its point of origin from the subclavian artery is lateral to clavicular injury. Furthermore, its course behind the costal cartilages is quite medial to the clavicular fracture. GAS 697, 952, 961, 973, 978; GA 37
48
A 68-year-old female is examined by the senior resident in emergency medicine after her fall on a wet bathroom fl oor in the shopping center. Physical examination reveals a posterior displacement of the left distal wrist and hand. Radiographic examination reveals an oblique fracture of the radius. Which of the following is the most likely fracture involved in this case? ⃣ A. Colles’ fracture ⃣ B. Scaphoid fracture ⃣ C. Bennett’s fracture ⃣ D. Volkmann’s ischemic contracture ⃣ E. Boxer’s fracture
A. Colles’ fracture is a fracture of the distal end of the radius. The proximal portion of the radius is displaced anteriorly, with the distal bone fragment projecting posteriorly. The displacement of the radius from the wrist often gives the appearance of a dinner fork, thus a Colles’ fracture is often referred to as a “dinner fork” deformity. A scaphoid fracture results from a fracture of the scaphoid bone and would thus not cause displace- ment of the radius. This fracture usually occurs at the narrow aspect (“waist”) of the scaphoid bone. Bennett’s and boxer’s fractures both result from fractures of the metacarpals (fi rst and fi fth, respectively). Volkmann’s ischemic contracture is a muscular deformity that can follow a supracondylar fracture of the humerus, with arterial laceration into the fl exor compartment of the forearm. Ischemia and muscle contracture, with extreme pain, accompany this fracture. GAS 734; GA 392
49
A 34-year-old female skier was taken by ambulance to the hospital after she struck a tree on the ski slope. Imaging gives evidence of a shoulder separation. Which of the following typically occurs in this kind of injury? ⃣ A. Displacement of the head of the humerus from the glenoid cavity ⃣ B. Partial or complete tearing of the coracoclavicular ligament ⃣ C. Partial or complete tearing of the coracoacromial ligament ⃣ D. Rupture of the transverse scapular ligament ⃣ E. Disruption of the glenoid labrum
B. In shoulder separation, either or both the acromioclavicular and coracoclavicular ligaments can be partially or completely torn through. The acromioclavicular joint can be interrupted and the distal end of the clavicle may deviate upward in a complete separation, while the upper limb droops away inferiorly, causing a “step off” that can be palpated and sometimes observed. Displacement of the head of the humerus is shoulder dislocation, not separation. The coracoacromial ligament is not torn in separation (but it is sometimes used in the repair of the torn coracoclavicular ligament). Disruption of the glenoid labrum often accompanies shoulder dislocation. GAS 669; GA 354
50
A 22-year-old male construction worker is admitted to the hospital after he suffers a penetrating injury to his upper limb from a nail gun. Upon physical examination the patient is unable to fl ex the distal interphalangeal joints of digits 4 and 5. What is the most likely cause of his injury? ⃣ A. Trauma to the ulnar nerve near the trochlea ⃣ B. Trauma to the ulnar nerve at the wrist ⃣ C. Median nerve damage proximal to the pronator teres ⃣ D. Median nerve damage at the wrist ⃣ E. Trauma to spinal nerve root C8
A. The nail was fi red explosively from the nail gun and then pierced the ulnar nerve near the coronoid process of the ulna trochlea of the humerus. Paralysis of the medial half of the fl exor digitorum profundus would result (among other signifi cant defi - cits), with loss of fl exion of the distal interphalangeal joints of digits 4 and 5. Ulnar trauma at the wrist would not affect the interphalangeal joints, although it would cause paralysis of interossei, hypothenar muscles, etc. Median nerve damage proximal to the pronator teres would affect proximal interphalangeal joint fl exion and distal interphalangeal joint fl exion of digits 2 and 3 as well as thumb fl exion. Median nerve injury at the wrist would cause loss of thenar muscles but not long fl exors of the fi ngers. Trauma to spinal nerve root C8 would affect all long fi nger fl exors. GAS 729, 771; GA 376, 390
51
The shoulder of a 44-year-old deer hunter had been penetrated by a bolt released from a crossbow. The bolt had transected the axillary artery just beyond the origin of the subscapular artery. A compress is placed on the wound with deep pressure. After a clamp is placed on the bleeding artery, thought is given to the anatomy of the vessel. What collateral arterial pathways are available to bypass the site of injury? ⃣ A. Suprascapular with circumfl ex scapular artery ⃣ B. Dorsal scapular with thoracodorsal artery ⃣ C. Posterior humeral circumfl ex artery with deep brachial artery ⃣ D. Lateral thoracic with brachial artery ⃣ E. Supreme thoracic artery with thoracoacromial artery
C. The injury has occurred just beyond the third part of the axillary artery. The only collateral arterial channel between the third part of the axillary artery and the brachial artery is that between the posterior humeral circumfl ex and the ascending branch of the profunda brachii—and this anastomotic path is often inadequate to supply the arterial needs of the limb. The posterior humeral circumfl ex arises from the third part of the axillary artery. It typically anastomoses with a variably small, ascending branch of the profunda brachii branch of the brachial artery. The suprascapular artery anastomoses with the circumfl ex scapular deep to the infraspinatus. The dorsal scapular artery (passing beneath the medial border of the scapula) has no anastomosis with thoracodorsal within the scope of the injury. The lateral thoracic artery has no anastomoses with the brachial artery. The supreme thoracic artery (from fi rst part of axillary) has no helpful anastomoses with the thoracoacromial (second part of axillary). GAS 683, 696, 697; GA 368
52
``` A 17-year-old male suffered the most common of fractures of the carpal bones when he fell on his outstretched hand. Which bone would this be? ⃣ A. Trapezium ⃣ B. Lunate UPPER LIMB 177 ⃣ C. Pisiform ⃣ D. Hamate ⃣ E. Scaphoid ```
E. The scaphoid (or the older term, navicular) bone is the most commonly fractured carpal bone. GAS 752-754; GA 392, 394, 422
53
A 54-year-old male cotton farmer visits the outpatient clinic because of a penetrating injury to his forearm with a baling hook. After the limb is anesthetized, the site of the wound is opened and fl ushed thoroughly to remove all debris. The patient is not able to oppose the tip of the thumb to the tip of the index fi nger, as in making the OK sign. He is able to touch the tips of the ring and little fi ngers to the pad of his thumb. What nerve has most likely been injured? ⃣ A. Median ⃣ B. Posterior interosseous ⃣ C. Radial ⃣ D. Recurrent median ⃣ E. Anterior interosseous
E. The anterior interosseous nerve is a branch of the median nerve that supplies the fl exor pollicis longus, the lateral half of the fl exor digitorum profundus, and the pronator quadratus. If it is injured, fl exion of the interphalangeal joint of the thumb will be compromised. The median nerve gives rise to the anterior interosseous nerve but is not a direct enough answer as injury to it would result in more widespread effects. The posterior interosseous nerve supplies extensors in the forearm, not fl exors. The radial nerve gives rise to the posterior interosseous nerve and is not associated with the anterior interosseous nerve; therefore, it would not have any effect on the fl exors of the forearm. The recurrent median nerve is also a branch of the median nerve but supplies the thenar eminence muscles, and its injury would result in problems with opposable motion of the thumb. GAS 743, 751; GA 400
54
Endoscopic examination of the shoulder of a 62-year-old female clearly demonstrated erosion of the tendon within the glenohumeral joint. What tendon was this? ⃣ A. Glenohumeral ⃣ B. Long head of triceps ⃣ C. Long head of biceps ⃣ D. Infraspinatus ⃣ E. Coracobrachialis
C. The tendon of the long head of the biceps brachii muscle passes through the glenohumeral joint, surrounded by synovial membrane. The glenohumeral is a ligament that attaches to the glenoid labrum. The long head of the triceps arises from the infraglenoid tubercle, beneath the glenoid fossa. The infraspinatus tendon passes posterior to the head of the humerus to insert on the greater tubercle. The coracobrachialis arises from the coracoid process and inserts on the humerus. GAS 694, 715-716, 735; GA 370
55
The orthopedic surgeon exposed the muscle in the supraspinous fossa so that she could move it laterally, in repair of an injured rotator cuff. As she refl ected the muscle from its bed, an artery was exposed crossing the ligament that bridges the notch in the superior border of the scapula. What artery was this? ⃣ A. Subscapular ⃣ B. Transverse cervical ⃣ C. Dorsal scapular ⃣ D. Posterior humeral circumfl ex ⃣ E. Suprascapular
E. The suprascapular artery passes over, and the suprascapular nerve passes under, the superior transverse scapular ligament. This ligament bridges the suprascapular notch in the upper border of the scapula. The artery and nerve then pass deep to the supraspinatus muscle, thereafter supplying it and then passing through the spinoglenoid notch to supply the infraspinatus. The subscapular artery is a branch of the third part of the axillary artery; it divides into circumfl ex scapular and thoracodorsal branches. The transverse cervical artery courses anterior to this site. The dorsal scapular artery and nerve pass deep to the medial border of the scapula. The posterior humeral circumfl ex branch of the axillary artery passes through the quadrangular space with the axillary nerve. GAS 696, 697; GA 366, 368
56
A 61-year-old man was hit by the cricket bat in the midhumeral region of his left arm. Physical examination reveals an inability to extend the wrist and loss of sensation on a small area of skin on the dorsum of the hand proximal to the fi rst two fi ngers. What nerve supplies this specifi c region of the hand? ⃣ A. Radial ⃣ B. Posterior interosseous ⃣ C. Lateral antebrachial cutaneous ⃣ D. Medial antebrachial cutaneous ⃣ E. Dorsal cutaneous of ulnar
A. The patient has suffered injury to the radial nerve in the midhumeral region. The nerve that provides sensation to the dorsum of the hand proximal to the thumb and index fi nger is the superfi cial branch of the radial nerve. The posterior interosseous nerve supplies a strip of skin on the back of the forearm and wrist extensors. The lateral antebrachial cutaneous nerve is a continuation of the musculocutaneous nerve and supplies the lateral side of the forearm. The medial antebrachial cutaneous is a direct branch of the medial cord and supplies skin of the medial side of the forearm. The dorsal cutaneous branch of the ulnar nerve supplies the medial side of the dorsum of the hand. GAS 772-774; GA 417
57
A 45-year-old woman is admitted to the hospital with neck pain. An MRI examination reveals a herniated disk in the cervical region. Physical examination reveals weakness in wrist extension and paraesthesia on the back of her arm and forearm. Which of the following spinal nerves is most likely injured? ⃣ A. C5 ⃣ B. C6 ⃣ C. C7 ⃣ D. C8 ⃣ E. T1
C. The seventh cervical nerve makes a major contribution to the radial nerve, and this nerve is the prime mover in wrist extension. The dermatome of C7 is in the region described. GAS 700-709; GA 361, 369-371
58
A 22-year-old male football player suffered a wrist injury while falling with force on his outstretched hand. When the anatomic snuffbox is exposed in surgery, an artery is visualized crossing the fractured bone that provides a fl oor for this space. What artery was visualized? ⃣ A. Ulnar ⃣ B. Radial ⃣ C. Anterior interosseous ⃣ D. Posterior interosseous ⃣ E. Deep palmar arch
B. As the radial artery passes from the ventral surface of the wrist to the dorsum, it crosses through the anatomic snuffbox, passing over the scaphoid bone. The ulnar artery at the wrist is located on the medial side of the wrist, passing from beneath the fl exor carpi ulnaris to reach Guyon’s canal between the pisiform bone and the fl exor retinaculum. The anterior interosseous and posterior interosseous arteries arise from the common interosseous branch of the ulnar artery and pass proximal to distal in the forearm between the radius and ulna, in the fl exor and extensor compartments, respectively. The deep palmar branch of the ulnar artery passes between the two heads of the adductor pollicis to anastomose with the radial artery in the palm. GAS 759; GA 422
59
The right shoulder of a 78-year-old female had become increasingly painful over the past year. Abduction of the right arm caused her to wince from the discomfort. Palpation of the deltoid muscle by the physician produced exquisite pain. Imaging studies reveal intermuscular infl ammation extending over the head of the humerus. What structure was infl amed? ⃣ A. Subscapular bursa ⃣ B. Infraspinatus muscle ⃣ C. Glenohumeral joint cavity ⃣ D. Subacromial bursa ⃣ E. Teres minor muscle
D. The patient is suffering from subacromial or subdeltoid bursitis. (If the pain on palpation is less when the arm has been elevated to the horizontal, the bursitis may be thought of as being more subacromial, that is, associated more with the supraspinatus tendon perhaps, for such a bursa may be drawn back under the acromion when the limb is abducted.) The subscapular bursa, beneath the subscapularis muscle, would not present as superfi cial pain. It can communicate with the glenohumeral joint cavity. Infl ammation or arthritic changes within the glenohumeral joint present as more generalized shoulder pain than that present here. The teres minor muscle and tendon are located inferior to the point of marked discomfort. GAS 670; GA 358
60
A 55-year-old male metallurgist had been diagnosed with carpal tunnel syndrome. To begin the operation, an anesthetic injection into his axillary sheath was used instead of a general anesthesia. From which of the following structures does the axillary sheath take origin? ⃣ A. Superfi cial fascia of the neck ⃣ B. Superfi cial cervical investing fascia ⃣ C. Buccopharyngeal fascia ⃣ D. Clavipectoral fascia ⃣ E. Prevertebral fascia
E. The axillary sheath is a fascial continuation of the prevertebral layer of the deep cervical fascia extending into the axilla. It encloses the nerves of the neurovascular bundle of the upper limb. Superfi cial fascia is loose connective tissue between the dermis and the deep investing fascia and contains fat, cutaneous vessels, nerves, lymphatics, and glands. The buccopharyngeal fascia covers the buccinator muscles and the pharynx mingles with the pretrachial fascia. The clavipectoral muscle invests the clavicle and pectoralis minor muscle. The axillary fascia is continuous with the pectoral and latissimus dorsi fascia and forms the hollow of the armpit. GAS 700-709; GA 361, 369-371
61
A 45-year-old woman is admitted to the hospital with neck pain. A CT scan reveals a tumor in the left side of her oral cavity. The tumor and related tissues are removed with a radical neck surgical procedure. Two months postoperatively the patient’s left shoulder droops quite noticeably. Physical examination reveals distinct weakness in turning her head to the right and impairment of abduction of her left upper limb to the level of the shoulder. Which of the following structures was most likely injured during the radical neck surgery? ⃣ A. Suprascapular nerve ⃣ B. Long thoracic nerve ⃣ C. Spinal accessory nerve ⃣ D. The junction of spinal nerves C5 and C6 of the brachial plexus ⃣ E. Radial nerve
C. The spinal accessory nerve arises from the ventral rootlets of C1 to C4 and ascends through the foramen magnum to then exit the cranial cavity through the jugular foramen. It innervates the sternocleidomastoid and trapezius muscles, which function in head rotation and raising of the shoulders. The suprascapular nerve receives fi bers from C5-6 (occasionally from C4 if the plexus is “prefi xed”) and innervates the supraspinatus muscle, which is responsible for the fi rst 15 ° of arm abduction. Erb point of the brachial plexus is at the union of C5-6 spinal nerves. The long thoracic nerve arises from plexus routes C5, 6, and 7, and supplies the serratus anterior. GAS 700-706; GA 361, 369-371
62
A 23-year-old male basketball player is admitted to the hospital after injuring his shoulder during a game. Physical and radiographic examinations reveal total separation of the shoulder ( Fig. 6-4 ). Which of the following structures has most likely been torn? ⃣ A. Glenohumeral ligament ⃣ B. Coracoacromial ligament ⃣ C. Tendon of long head of biceps brachii ⃣ D. Acromioclavicular ligament ⃣ E. Transverse scapular ligament
D. The acromioclavicular ligament connects the clavicle to the coracoid process of the scapula. Separation of the shoulder (dislocation of the acromioclavicular [AC] joint) is associated with damage to the acromioclavicar ligament (capsule of the AC joint) and, in more severe injuries, disruption of the coracoclavicular ligaments (conoid and trapezoid portions). The glenohumeral ligament may be injured by an anterior dislocation of the humerus but is not likely to be injured by a separated shoulder. The coracoacromial ligament, transverse scapular ligament, and tendon of the long head of triceps brachii are not likely to be injured by separation of the shoulder. GAS 669; GA 354, 363
63
A 35-year-old male body builder has enlarged his shoulder muscles to such a degree that the size of the quadrangular space is greatly reduced. Which of the following structures would most likely be compressed in this condition? ⃣ A. Axillary nerve ⃣ B. Anterior humeral circumfl ex artery ⃣ C. Cephalic vein ⃣ D. Radial nerve ⃣ E. Subscapular artery
A. The quadrangular space is bordered medially by the long head of the triceps, laterally by the surgical neck of the humerus, superiorly by the teres minor and subscapularis muscles, and inferiorly by the teres major muscle. Both the axillary nerve and posterior humeral circumfl ex vessels traverse this space. The other structures listed are not contained within the quadrangular space. The cephalic vein is located in the deltopectoral triangle, and the radial nerve is located in the triangular interval. GAS 700-709; GA 361, 369-371
64
A 43-year-old woman visits the outpatient clinic with a neurologic problem. Diagnostically, she cannot hold a piece of paper between her thumb and the lateral side of her index fi nger without fl exing the distal joint of her thumb. This is a positive Froment sign and a diagnosis of ulnar neuropathy. Weakness of which specifi c muscle causes this sign to appear? ⃣ A. Flexor pollicis longus ⃣ B. Adductor pollicis ⃣ C. Flexor digiti minimi ⃣ D. Flexor carpi radialis ⃣ E. Extensor indicis
B. Froment’s sign is positive for ulnar nerve palsy. More specifi cally it tests the action of the adductor pollicis muscle. The patient is asked to hold a sheet of paper between the thumb and a fl at palm. The fl exor pollicis longus is innervated by the anterior interosseous branch of the median nerve. The fl exor digiti minimi is innervated by the deep branch of the ulnar nerve and would not be used to hold a sheet of paper between the thumb and palm. The fl exor carpi radialis is innervated by the median nerve, and the extensor indicis is innervated by the radial nerve. GAS 763-765; GA 410, 412
65
A 48-year-old female piano player visited the outpatient clinic with numbness and tingling in her left hand. A diagnosis was made of nerve compression in the carpal tunnel, and the patient underwent an endoscopic nerve release. Two weeks postoperatively the patient complained of a profound weakness in the thumb, with loss of thumb opposition. The sensation to the hand, however, was unaffected. Which of the following nerves was injured during the operation? ⃣ A. The fi rst common digital branch of the median nerve ⃣ B. The second common digital branch of the median nerve ⃣ C. Recurrent branch of median nerve ⃣ D. Deep branch of the ulnar nerve ⃣ E. Anterior interosseus nerve
C. The recurrent branch of the median nerve innervates the thenar muscles (opponens pollicis, abductor pollicis brevis, and fl exor pollicis brevis) and is not responsible for any cutaneous innervation. Damage to the palmar cutaneous branches of the median nerve or to the ulnar nerve would not cause weakness of opposition of the thumb for they are principally sensory in function. The deep branch of the ulnar nerve supplies the hypothenar muscles, adductor and abductor muscles of digits 2–5, and does not innervate the abductor policis brevis. GAS 770-771; GA 414
66
A 19-year-old male had suffered a deep laceration to an upper limb when he stumbled and fell on a broken bottle. On examination of hand function it is observed that he is able to extend the metacarpophalangeal joints of all his fi ngers in the affected limb. He cannot extend the interphalangeal joints of the fourth and fi fth digits, and extension of the interphalangeal joints of the second and third digits is very weak. There is no apparent sensory defi cit in the hand. Which of the following nerves has most likely been injured? ⃣ A. Radial nerve at the elbow ⃣ B. Median nerve at the wrist ⃣ C. Ulnar nerve in midforearm ⃣ D. Deep branch of ulnar nerve ⃣ E. Recurrent branch of the median nerve
D. Injury to the deep branch of the ulnar nerve results in paralysis of all interosseous muscles and the lumbrical muscles of digits 4 and 5. Extension of the metacarpophalangeal joints is intact, a function of the radial nerve. Interphalangeal extension of digits 4 and 5 is absent, due to the loss of all interosseous muscle and the lumbricals of digits 4 and 5. Some weak interphalangeal joint extension is still present in digits 2 and 3 because the lumbricals of these two fi ngers are innervated by the median nerve. The radial nerve and the median nerve appear to be intact in this case. If the ulnar nerve were injured in the midforearm region, there would be sensory loss in the palm and digits 4 and 5 and on the dorsum of the hand. The recurrent branch of the median nerve supplies the thenar muscles; it does not supply lumbricals. Moreover, paralysis of this nerve would have no effect on the interphalangeal joints. GAS 770-771; GA 415
67
A 41-year-old woman is scheduled for a latissimus dorsi muscle fl ap to cosmetically augment the site of her absent left breast, postmastectomy. Part of the latissimus dorsi muscle is advanced to the anterior thoracic wall, based upon arterial supply provided in part by the artery that passes through the triangular space of the axilla. What artery is forming the vascular base of this fl ap? ⃣ A. Circumfl ex scapular artery ⃣ B. Dorsal scapular artery ⃣ C. Transverse cervical artery ⃣ D. Lateral thoracic artery ⃣ E. Thoracoacromial artery
A. The circumfl ex scapular artery passes through the triangular space after arising from the subscapular artery. It provides superfi cial branches to the overlying latissimus dorsi, whereas its deep portion passes into the infraspinous fossa to anastomose with the suprascapular artery. The dorsal scapular artery passes between the roots of the brachial plexus and then deep to the medial border of the scapula. The transverse cervical artery arises from the thyrocervical trunk at the root of the neck and can provide origin for a dorsal scapular branch. The lateral thoracic and thoracoacromial arteries are branches of the second part of the axillary artery and provide no supply to the latissimus dorsi. GAS 683, 684, 697; GA 368
68
A 31-year-old male hockey player fell on his elbow and is admitted to the emergency department. Radiographic examination reveals a fracture of the surgical neck of the humerus, producing an elevation and adduction of the distal fragment. Which of the following muscles would most likely cause the adduction of the distal fragment? ⃣ A. Brachialis ⃣ B. Teres minor ⃣ C. Pectoralis major ⃣ D. Supraspinatus ⃣ E. Pectoralis minor
C. The surgical neck of the humerus is a typical site of fractures. The fracture line lies above the insertions of the pectoralis major, teres major, and latissimus dorsi muscles. The supraspinatus muscle abducts the proximal fragment, whereas the distal fragment is elevated and adducted. The elevation results from contraction of the deltoid, biceps brachii, and coracobrachialis muscles. The adduction is due to the action of pectoralis major, teres major, and latissimus dorsi. GAS 139-140, 659, 686-687; GA 374, 492
69
A 74-year-old woman is admitted to the emergency department after stumbling over her pet dog. Radiographic examination reveals a fracture of the upper third of the right radius, with the distal fragment of the radius and hand pronated. The proximal end of the fractured radius deviates laterally. Which of the following muscles is primarily responsible for the lateral deviation? ⃣ A. Pronator teres ⃣ B. Supinator ⃣ C. Pronator quadratus ⃣ D. Brachioradialis ⃣ E. Brachialis
B. The fracture line of the upper third of the radius lies between the bony attachments of the supinator and the pronator teres muscles. The distal radial fragment and hand are pronated due to unopposed contraction of pronator teres and pronator quadratus muscles. The proximal fragment deviates laterally by the unopposed contraction of the supinator muscle. The brachioradialis inserts distally on the radius. The brachialis inserts on the coronoid process of the ulna and would not be involved in the lateral deviation of the radius. GAS 736, 747-749; GA 390
70
A 12-year-old male had received a laceration in the palmar surface of the wrist while playing with a very sharp knife. The cut ends of a tendon could be seen within the wound in the exact midline of the wrist. Which tendon lies in this position in most people? ⃣ A. Palmaris longus ⃣ B. Flexor carpi radialis ⃣ C. Abductor pollicis longus ⃣ D. Flexor carpi ulnaris ⃣ E. Flexor pollicis longus
A. The palmaris longus passes along the midline of the fl exor surface of the forearm. The fl exor carpi radialis is seen in the lateral portion of the forearm superfi cially, passing over the trapezium to insert at the base of the second metacarpal. The abductor pollicis longus tendon is laterally located in the wrist, where it helps form the lateral border of the anatomic snuffbox. The fl exor carpi ulnaris tendon can be seen and palpated on the medial side of the wrist ventrally. The fl exor pollicis longus tendon passes deep through the carpal tunnel. GAS 737-739; GA 374, 390, 398
71
A 22-year-old male medical student was seen in the emergency department with a complaint of pain in his hand. He confessed that he had hit a vending machine in the hospital when he did not receive his soft drink after inserting money twice. The medial side of the dorsum of the hand was quite swollen, and one of his knuckles could not be seen when he “made a fi st.” The physician made a diagnosis of a “boxer’s fracture.” What was the nature of the impatient student’s injury? ⃣ A. Fracture of the styloid process of the ulna ⃣ B. Fracture of the neck of the fi fth metacarpal ⃣ C. Colles’ fracture of the radius ⃣ D. Smith’s fracture of the radius ⃣ E. Bennett’s fracture of the thumb
B. The student had broken the neck of the fi fth metacarpal when hitting the machine with his fi st. This is the more common type of “boxer’s fracture.” Neither a fracture of the ulnar styloid nor a Colles’ fracture nor a Smith fracture of the distal radius would present with the absence of a knuckle as observed here. Bennett’s fracture involves dislocation of the carpometacarpal joint of the thumb. Indications are that the injury is on the medial side of the hand, not the wrist, nor the lateral side of the hand or wrist. GAS 769; GA 392, 394
72
Fine motor function in the right hand of a 14- year-old female with scoliosis since birth appeared to be quite reduced, including thumb opposition, abduction and adduction of the digits, and interphalangeal joint extension. Radiography confi rmed that her severe scoliosis was causing marked elevation of the right fi rst rib. Long fl exor muscles of the hand and long extensors of the wrist appear to be functioning within normal limits. There is notable anesthesia of the skin on the medial side of the forearm; otherwise, sensory function in the limb is intact. Which of the following neural structures is most likely impaired? ⃣ A. Median nerve ⃣ B. Middle trunk of the brachial plexus ⃣ C. Radial nerve ⃣ D. Lower trunk of the brachial plexus ⃣ E. T1 nerve root
E. Scoliosis (severe, lateral curvature of the spine) in the patient is causing compression or stretching of the T1 spinal nerve root by the fi rst rib as the nerve ascends to join C8 and form the lower trunk of the brachial plexus. T1 provides sensation for the medial side of the forearm, via the medial antebrachial cutaneous nerve from the medial cord of the brachial plexus. T1 is the principal source of motor supply to all of the intrinsic muscles in the palm. Its dysfunction affects all fi ne motor movements of the digits. Long fl exors of the fi ngers are intact; therefore, the median nerve and ulnar nerve are not at fault. The extensors of the wrist are functional; therefore, the radial nerve is not paralyzed. The only sensory disturbance is that of the T1 dermatome. GAS 700-709; GA 361, 369-371
73
A 23-year-old female had a painful injury to her hand in a dry ski-slope competition, in which she fell and caught her thumb in the matting. Radiographic and physical examinations reveal rupture of the ulnar collateral ligament of the metacarpophalangeal joint of the thumb. Lidocaine is injected into the area to relieve the pain, and she is scheduled for a surgical repair. From which of the following clinical problems is she suffering? ⃣ A. De Quervain’s syndrome ⃣ B. Navicular bone fracture ⃣ C. Boxer’s thumb ⃣ D. Gamekeeper’s thumb ⃣ E. Bennett’s thumb
D. Interestingly, “gamekeeper’s thumb” was a term coined because this injury was most commonly associated with Scottish gamekeepers who, it is said, killed small animals such as rabbits by breaking their necks between the ground and the gamekeeper’s thumb and index fi nger. The resulting valgus force on the abducted MCP joint caused injury to the ulnar collateral ligament. These days this injury is more commonly seen in skiers who land awkwardly with their hand braced on a ski pole, causing the valgus force on the thumb as is seen in this patient. Whereas the term “skier thumb” is sometimes used, “gamekeeper’s thumb” is still in common usage. GAS 755; GA 395
74
A 26-year-old male power lifter visits the outpatient clinic with a painful shoulder. Radiographic examination reveals tendinopathy of the long head of the biceps. Which of the following conditions will most likely be present during physical examination? ⃣ A. Pain is felt in the anterior shoulder during forced contraction. ⃣ B. Pain is felt in the lateral shoulder during forced contraction. ⃣ C. Pain is felt during abduction and fl exion of the shoulder joint. ⃣ D. Pain is felt during extension and adduction of the shoulder joint. ⃣ E. Pain is felt in the lateral shoulder during fl exion of the shoulder joint.
A. The long head of the biceps assists in shoulder fl exion and during a tendinopathy would cause pain in the anterior compartment of the shoulder, where it originates at the supraglenoid tubercle. Also, forced contraction would cause more of a greater tension force on the tendon. GAS 694, 715-716; GA 370
75
A 43-year-old female tennis player visits the outpatient clinic with pain over the right lateral epicondyle of her elbow. Physical examination reveals that the patient has lateral epicondylitis. Which of the following tests should be performed during physical examination to confi rm the diagnosis? ⃣ A. Nerve conduction studies ⃣ B. Evaluation of pain experienced during fl exion and extension of the elbow joint ⃣ C. Observing the presence of pain when the wrist is extended against resistance ⃣ D. Observing the presence of numbness and tingling in the ring and little fi ngers when the wrist is fl exed against resistance E. ⃣ Evaluation of pain felt over the styloid process of radius during brachioradialis contraction
C. The common extensor tendon originates from the lateral epicondyle, and infl ammation of this tendon is lateral epicondylitis, nicknamed “tennis elbow” because the tendon is often irritated during the backhand stroke in tennis. Because the extensors of the wrist originate as part of the common extensor tendon, extension of the wrist will exacerbate the pain of lateral epicondylitis. GAS 691; GA 380
76
A male skier had a painful fall against a rocky ledge. Radiographic fi ndings revealed a hairline fracture of the surgical neck of the humerus. The thirdyear medical student assigned to this patient was asked to determine whether there was injury to the nerve associated with the area of injury. Which of the following tests would be best for checking the status of the nerve? ⃣ A. Have the patient abduct the limb while holding a 10-lb weight. ⃣ B. Have the patient shrug the shoulders. ⃣ C. Test for presence of skin sensation over the lateral side of the shoulder. ⃣ D. Test for normal sensation over the medial skin of the axilla. ⃣ E. Have the patient push against an immovable object like a wall and assess the position of the scapula.
C. The axillary nerve passes dorsally around the surgical neck of the humerus (accompanied by the posterior humeral circumfl ex artery) and can be injured when the humerus is fractured at that location. The axillary nerve provides sensation to the skin over the upper, lateral aspect of the shoulder. Therefore, although the patient might not be able to abduct the arm because of the injury, a simple test of skin sensation can indicate whether there is associated nerve injury of the axillary nerve. Shrugging the shoulders can help assess trapezius function, thereby testing the spinal accessory nerve. Intact sensation of the skin on the medial aspect of the axilla or arm is an indication that the radial or intercostobrachial nerves are functional. Pushing against an immovable object tests the serratus anterior muscle and the long thoracic nerve. GAS 667; GA 355
77
A 27-year-old male had lost much of the soft tissue on the dorsum of his left hand in a motorcycle crash. Imaging studies show no other upper limb injuries. Because the left extensor carpi radialis longus and brevis tendons were lost, it was decided to replace those tendons with the palmaris longus tendons from both forearms because of those tendons’ convenient location and relative unimportance. Postoperatively it is found that sensation is absent in both hands on the lateral palm and palmar surfaces of the fi rst three digits; there is also paralysis of thumb opposition. What is the most likely cause of the sensory defi cit and motor loss in both thumbs? ⃣ A. Bilateral loss of spinal nerve T1 with fractures of fi rst rib bilaterally ⃣ B. Lower plexus (lower trunk) trauma ⃣ C. Dupuytren contracture ⃣ D. Left radial nerve injury in the posterior compartment of the forearm ⃣ E. The palmaris longus was absent bilaterally; the nerve beneath it looked like a tendon.
E. The surgeon took the distal segments of the median nerves from both forearms, mistakenly believing them to be palmaris longus tendons. Both of the structures lie in the midline of the ventral surface of the distal forearm and are often of similar appearance in color and diameter. The nerve is located deep to the tendon, when the tendon is present, but when the tendon is absent, the nerve appears to be where the tendon belongs. There is no evidence of rib fractures; even so, a fractured rib would not explain loss of sensation on the lateral portion of the palm. Lower plexus trauma (C8, T1) would result in paralysis of forearm fl exor muscles and all intrinsic hand muscles and sensory loss over the medial dorsum of the hand, in addition to palmar sensory loss. Dupuytren’s contracture is a fl exion contracture of (usually) digits 4 and 5 from connective tissue disease in the palm. Radial nerve injury in the posterior forearm would affect metacarpophalangeal joint extension, thumb extension, etc., not palmar disturbances. GAS 743-744; GA 400
78
A 15-year-old male received a shotgun wound to the ventral surface of the upper limb. Upon examination it is quickly observed that the patient exhibits a complete clawhand but can extend his wrist. What is the nature of this patient’s injury? A. ⃣ The ulnar nerve has been severed at the wrist. ⃣ B. The median nerve has been injured in the carpal tunnel. ⃣ C. The median and ulnar nerves are damaged at the wrist. ⃣ D. The median and ulnar nerves have been injured at the elbow region. ⃣ E. The median, ulnar, and radial nerves have been injured at midhumerus.
C. Trauma both to the median and ulnar nerves at the wrist results in total clawing of the fi ngers. The metacarpophalangeal joints of all digits are extended by the unopposed extensors because the radial nerve is intact. All interossei and lumbricals are paralyzed because the deep branch of the ulnar nerve supplies all of the interossei; lumbricals I and II are paralyzed, for they are innervated by the median nerve; lumbricals III and IV are paralyzed, for they receive supply from the deep ulnar nerve. The interossei and lumbricals are responsible for extension of the interphalangeal joints. When they are paralyzed, the long fl exor tendons pull the fi ngers into a position of fl exion, completing the “claw” appearance. If the median nerve were intact, the clawing would be less noticeable in the index and long fi nger because the two lumbricals would still be capable of some degree of extension of those interphalangeal joints. If the median nerve alone is injured in the carpal tunnel, there would be loss of thenar opposition but not clawing. If the median and ulnar nerves are both transected at the elbow, the hand appears totally fl at because of the loss of long fl exors, in addition to intrinsic paralysis. GAS 726; GA 400
79
A 68-year-old woman fell when she missed the last step from her motor home. Radiographic examination at the local medical care center reveals a fracture of the distal radius. The distal fragment of the radius is angled forward. What name is commonly applied to this type of injury? ⃣ A. Colles’ fracture ⃣ B. Scaphoid fracture ⃣ C. Bennett’s fracture ⃣ D. Smith’s fracture ⃣ E. Boxer’s fracture
D. Colles’ fracture is a fracture of the distal radius with the distal fragment displaced dorsally. Smith’s fracture involves the distal fragment displaced in a volar direction. Smith’s fracture is sometimes referred to as a reverse Colles’ fracture. GAS 756; GA 392
80
It was reported by the sports media that the outstanding 27-year-old shortstop for the New York team would miss a number of baseball games. He was hit on a fi ngertip while attempting to catch a ball barehanded. A tendon had been torn. The team doctor commented that the ballplayer could not straighten the last joint of the long fi nger of his right hand, and the fi nger would require surgery. From what injury did the ballplayer suffer? ⃣ A. Clawhand deformity ⃣ B. Boutonnière deformity ⃣ C. Swan-neck deformity ⃣ D. Dupuytren contracture ⃣ E. Mallet fi nger
E. The extensor tendons of the fi ngers insert distally on the distal phalanx of each digit. If the tendon is avulsed, or the proximal part of the distal phalanx is detached, the distal interphalangeal joint is pulled into total fl exion by the unopposed fl exor digitorum profundus. This result gives the digit the appearance of a mallet. In boutonnière deformity, the central portion of the extensor tendon expansion is torn over the PIP joint, allowing the tendon to move palmarward, causing the tendon to act as a fl exor of the PIP joint. This causes the DIP joint to be hyperextended. Swanneck deformity involves slight fl exion of MCP joints, hyperextension of PIP joints, and slight fl exion of DIP joints. This condition results most often from shortening of the tendons of intrinsic muscles, as in rheumatoid arthritis. Dupuytren contracture results from connective tissue disorder in the palm, usually causing irreversible fl exion of digits 4 and 5. Clawhand occurs with lesions to the median and ulnar nerves at the wrist. In this clinical problem all intrinsic muscles are paralyzed, including the extensors of the interphalangeal joints. The MCP joint extensors, supplied by the radial nerve, and the long fl exors of the fi ngers, supplied more proximally in the forearm by the median and ulnar nerves, are intact and are unopposed, pulling the fi ngers into the “claw” appearance. GAS 760; GA 392, 394, 422
81
A 31-year-old female fi gure skater is examined in the emergency department following an injury that forced her to withdraw from competition. When her male partner missed catching her properly from an overhead position, he grasped her powerfully, but awkwardly, by the forearm. Clinical examination demonstrated a positive Ochsner test, inability to fl ex the distal interphalangeal joint of the index fi nger on clasping the hands. In addition, she is unable to fl ex the terminal phalanx of the thumb and has loss of sensation over the thenar half of the hand. What is the most likely nature of her injury? ⃣ A. Median nerve injured within the cubital fossa ⃣ B. Anterior interosseous nerve injury at the pronator teres ⃣ C. Radial nerve injury at its entrance into the posterior forearm compartment ⃣ D. Median nerve injury at the proximal skin crease of the wrist E. ⃣ Ulnar nerve trauma halfway along the forearm
A. Because the median nerve is injured within the cubital fossa, the long fl exors are paralyzed, including the fl exor pollicis longus. Flexor pollicis longus would not be paralyzed if the median nerve were injured at the wrist. Lateral palm sensory loss con- fi rms median nerve injury. If only the anterior interosseous nerve were damaged, there would be no cutaneous sensory defi cit. The radial nerve supplies wrist extensors, long thumb abductor, and metacarpophalangeal joint extensors. The ulnar nerve does not supply sensation to the lateral palm. GAS 651, 729, 731; GA 400
82
A 19-year-old fell from a cliff when he was hiking in the mountains. He broke his fall by grasping a tree branch, but he suffered injury to the C8-T1 spinal nerve roots. Sensory tests would thereafter con- fi rm the nature of his neurologic injury by the sensory loss in the part of the limb supplied by which of the following? ⃣ A. Lower lateral brachial cutaneous nerve ⃣ B. Musculocutaneous nerve ⃣ C. Intercostobrachial nerve ⃣ D. Medial antebrachial cutaneous nerve ⃣ E. Median nerve
D. In a lesion of the lower trunk, or the C8 and T1 nerve roots, there is sensory loss on the medial forearm and the medial side of hand (dorsal and ventral). The medial cord is the extension of the lower trunk. The medial cord gives origin to the medial antebrachial cutaneous nerve, which supplies the T1 dermatome of the medial side of the antebrachium. The lower lateral brachial cutaneous nerve arises from the radial nerve, C5 and C6. The musculocutaneous nerve arises from the lateral cord, ending in the lateral antebrachial cutaneous nerve, with C5 and C6 dermatome fi bers. The intercostobrachial nerve is the lateral cutaneous branch of the T2 ventral primary ramus and supplies skin on the medial side of the arm. The median nerve distributes C6 and C7 sensory fi bers to the lateral part of the palm, thumb, index, long fi nger, and half of the ring fi nger. GAS 743-745, 771-772; GA 384
83
The mastectomy procedure on a 52-year-old female involved excision of the tumor and a removal of lymph nodes, including the pectoral, central axillary, and infraclavicular groups. Six months after her mastectomy, the patient complains to her personal physician of an unsightly deep hollow area inferior to the medial half of the clavicle, indicating a signifi cant area of muscle atrophy and loss. She states that the disfi gurement has taken place quite gradually since her mastectomy. Physical examination reveals no obvious motor or sensory defi cits. What was the most likely cause of the patient’s cosmetic problem? ⃣ A. Part of the pectoralis major muscle was cut and removed in the mastectomy. ⃣ B. The pectoralis minor muscle was removed entirely in the surgery. C. ⃣ A branch of the lateral pectoral nerve was cut. ⃣ D. The medial pectoral nerve was cut. ⃣ E. The lateral cord of the brachial plexus was injured.
C. The fi rst branch of the lateral pectoral nerve is typically the only source of motor supply to the clavicular head of the pectoralis major. If it is injured (as in this case of an iatrogenic injury when the infraclavicular nodes were removed), this part of the muscle undergoes atrophy, leaving an infraclavicular cosmetic defi cit. The remainder of the lateral pectoral nerve joins the medial pectoral nerve in a neural arch that provides motor supply to the remaining parts of the pectoralis major and the pectoralis minor. Physical examination reveals no obvious motor or sensory defi cits. Loss of the medial pectoral nerve would have no effect on the clavicular head of pectoralis major and might not be discernible. Injury to the lateral cord would lead to loss not only of all of the lateral pectoral nerve but also the musculocutaneous nerve, resulting in biceps and brachialis paralysis and lateral antebrachial sensory loss. GAS 688, 702, 706; GA 370
84
A 54-year-old female was found unconscious on the fl oor, apparently after a fall. She was admitted to the hospital, and during physical examination it was observed that she had absence of her brachioradialis refl ex. Which spinal nerve is primarily responsible for this refl ex in the majority of cases? ⃣ A. C5 ⃣ B. C6 ⃣ C. C7 ⃣ D. C8 ⃣ E. T1
B. The C6 spinal nerve is primarily responsible for the brachioradialis refl ex. C5 and C6 are both involved in the biceps brachii refl ex; C5 for motor, C6 for the sensory part of the refl ex arc; C7 is the key spinal nerve in the triceps refl ex. GAS 700-709; GA 361, 369-371
85
A 43-year-old man is admitted to the hospital, having suffered a whiplash injury when his compact auto was struck from behind by a sports utility vehicle. MRI examination reveals some herniation of a disk in the cervical region. Physical examination reveals that the patient has lost elbow extension; there is absence of his triceps refl ex and loss of extension of the metacarpophalangeal joints on the ipsilateral side. Which of the following spinal nerves is most likely affected? ⃣ A. C5 ⃣ B. C6 ⃣ C. C7 ⃣ D. C8 ⃣ E. T1
C. C7 is the main spinal nerve that contributes to the radial nerve and innervates the triceps. Absence of the triceps refl ex is usually indicative of a C7 radiculopathy or injury. GAS 700-709; GA 361, 369-371
86
A 29-year-old patient has a dislocated elbow in which the ulna and medial part of the distal humerus have become separated. What classifi cation of joint is normally formed between these two bones? ⃣ A. Trochoid ⃣ B. Ginglymus ⃣ C. Enarthrodial ⃣ D. Synarthrosis ⃣ E. Sellar
B. Ginglymus joint is the technical term to describe a hinge joint. It allows motion in one axis (fl exion and extension in the case of the elbow) and is therefore a uniaxial joint. The other types of joints listed allow motion in more than one axis. GAS 79-81; GA 372
87
A 45-year-old woman motorcyclist, propelled over the handlebars of her bike by an encounter with a rut in the road, lands on the point of one shoulder. The woman is taken by ambulance to the emergency department. During physical examination the arm appears swollen, pale, and cool. Any movement of the arm causes severe pain. Radiographic examination reveals a fracture and a large hematoma, leading to diagnosis of Volkmann’s ischemic contracture. At which of the following locations has the fracture most likely occurred? ⃣ A. Surgical neck of humerus ⃣ B. Radial groove of humerus ⃣ C. Supracondylar line of humerus ⃣ D. Olecranon ⃣ E. Lateral epicondyle
C. A fracture of the humerus just proximal to the epicondyles is called a supracondylar fracture. This is the most common cause of a Volkmann ischemic fracture. The sharp bony fragment often lacerates the brachial (or other) artery, with bleeding into the fl exor compartment. Diminution of arterial supply to the compartment results in the ischemia. Bleeding into the compartment causes greatly increased pressure, fi rst blocking venous outfl ow from the compartment, then reducing the arterial fl ow into the compartment. The ischemic muscles then undergo unrelieved contracture. A humeral fracture is sometimes placed in a cast from shoulder to wrist, often concealing the ischemia until major tissue loss occurs. Cold, insensate digits and great pain are warnings of this compartmental problem, demanding that the cast be removed and the compartment opened (“released”) for pressure reduction and vascular repair. Fracture of the surgical neck endangers the axillary nerve and posterior humeral circumfl ex artery, although not ischemic contracture. Fracture of the humerus in the spiral groove can injure the radial nerve and profunda brachii artery. Fracture of the olecranon does not result in Volkmann’s contracture, although the triceps brachii can displace the distal fractured fragment of the ulna. GAS 728; GA 355
88
A 55-year-old female choreographer had been treated in the emergency department after she fell from the stage into the orchestra pit. Radiographs revealed fracture of the styloid process of the ulna. Disruption of the triangular fi brocartilage complex is suspected. With which of the following bones does the ulna normally articulate at the wrist? ⃣ A. Triquetrum ⃣ B. Hamate ⃣ C. Radius and lunate ⃣ D. Radius ⃣ E. Pisiform and triquetrum
D. Normally, the distal part of the ulna articulates only with the radius at the distal radioulnar joint at the wrist, a joint that participates in pronation/ supination. The head of the ulna does not articulate with any of the carpal bones; instead, it is separated from the triquetrum and lunate bones by the triangular fi brocartilage complex between it and the radius. The pisiform articulates with the triquetrum. The carpal articulation of the radius is primarily that of the scaphoid (old name is navicular) bone. GAS 713, 714, 731-733; GA 391
89
A 67-year-old female had a bad fall while walking her dog the evening before. She states that she fell on her outstretched hand. Radiographs do not demonstrate any bony fractures. The clinician observes the following signs of neurologic injury: There is weakness of fl exion of her wrist in a medial direction, there is a loss of sensation on the medial side of the hand, and there is clawing of the fi ngers. Where is the most likely place of nerve trauma? ⃣ A. Behind the medial epicondyle ⃣ B. Between the pisiform bone and the fl exor retinaculum ⃣ C. Within the carpal tunnel ⃣ D. At the cubital fossa, between the ulnar and radial heads of origin of fl exor digitorum superfi cialis ⃣ E. At the radial neck, 1 cm distal to the humerocapitellar joint
A. The force of the woman’s fall on the outstretched hand was transmitted up through the forearm, sometimes resulting in dislocation of the olecranon at the elbow, putting traction on the ulnar nerve as it passes around the medial epicondyle of the humerus. Ulnar trauma at the elbow can cause weakness in medial fl exion (adduction) at the wrist, from loss of the fl exor carpi ulnaris. Ulnar nerve injury also results in sensory loss in the medial hand and paralysis of the interossei and medial two lumbricals, with clawing especially of digits 4 and 5. Injury of the ulnar nerve at the pisiform bone would not affect the fl exor carpi ulnaris, nor would it produce sensory loss on the dorsum of the hand because the dorsal cutaneous branch of the ulnar branches off proximal to the wrist. Carpal tunnel problems affect median nerve function, which is not indicated here. The ulnar nerve passes medial to the cubital fossa between the heads of the fl exor carpi ulnaris, not between the heads of the fl exor digitorum superfi cialis. Injuries at the radial neck affect the site of division of the radial nerve, and its paralysis would not result in the clinical problems seen in this patient. GAS 727-728; GA 386
90
An 18-year-old male suffered a signifi cant laceration through the skin and underlying tissues at the distal crease of the wrist. The medical student rotating through the emergency department suspected (correctly) that the ulnar nerve was cut completely through at this location. Which of the following would most likely occur? ⃣ A. The patient could not touch the tip of the thumb to the tips of the other digits. ⃣ B. There would be loss of sensation on the dorsum of the medial side of the hand. ⃣ C. The patient would be unable to fl ex the interphalangeal joints. ⃣ D. There would be decreased ability to extend the interphalangeal joints. ⃣ E. There would be no serious functional problem at all to the patient.
D. The interossei are the most important muscles in extension of the interphalangeal (IP) joints because of the manner of their insertion into the extensor expansion of the fi ngers, which passes dorsal to the transverse axes of these joints. The lumbrical muscles assist in IP extension, in addition to fl exing the metacarpophalangeal joints. Ulnar nerve injury at the wrist results in paralysis of all the interossei and the medial two lumbricals. Extensors of the MCP joints are innervated by the deep radial nerve. Unopposed extension of the MCP joints causes them to be held in extension whereas unopposed long fl exors of the fi ngers (supplied by median and ulnar nerves proximally in the forearm) cause them to be fl exed into the “claw” position. The lumbricals of digits 2 and 3 are still intact because they are supplied by the median nerve, so clawing is not seen as much on these digits. Loss of opposition would result from median or recurrent nerve paralysis. If the ulnar nerve is cut at the wrist, its dorsal cutaneous branch to the dorsum of the hand is unaffected. GAS 655, 755; GA 395
91
A 45-year-old man visits the outpatient clinic after a digit of his left hand was injured when a door was slammed on his hand. A superfi cial cut on his middle fi nger has been sutured, but functional defi cits are observed in the fi nger: The proximal interphalangeal joint is pulled into constant fl exion, whereas the distal interphalangeal joint is held in a position of hyperextension. What is the most likely diagnosis? ⃣ A. Mallet fi nger ⃣ B. Boutonnière deformity ⃣ C. Dupuytren contracture ⃣ D. Swan-neck deformity ⃣ E. Silver fork wrist deformity
B. In boutonnière deformity, the central portion of the extensor tendon expansion is torn over the PIP joint, allowing the tendon to move palmarward, causing the tendon to act as a fl exor of the PIP joint. This causes the DIP joint to be hyperextended. The tear in the extensor tendon is said to resemble a buttonhole (“boutonnière” in French), and the head of the proximal phalanx may stick through the hole. GAS 732-755; GA 395
92
A 67-year-old housepainter visits the outpatient clinic complaining that his hands are getting progressively worse, becoming more and more painful and losing their function. On physical examination of the hands, there is fl exion of the metacarpophalangeal joints, extension of the proximal interphalangeal joints, and slight fl exion of the distal interphalangeal joints. What is the most likely diagnosis? ⃣ A. Mallet fi nger ⃣ B. Boutonnière deformity ⃣ C. Dupuytren contracture ⃣ D. Swan-neck deformity ⃣ E. Silver fork wrist deformity
D. Swan-neck deformity involves slight fl exion of MCP joints, hyperextension of PIP joints, and slight fl exion of DIP joints. This condition results most often from shortening of the tendons of intrinsic muscles, as in rheumatoid arthritis. When asked to straighten the injured fi nger, the patient is unable to do so and the curvature of the fi nger somewhat resembles the neck of a swan. GAS 732-755; GA 395
93
Several weeks after surgical dissection of her left axilla for the removal of lymph nodes for staging and treatment of her breast cancer, a 32-year-old woman was told by her general physician that she had “winging” of her left scapula when she pushed against resistance during her physical examination. She told the physician that she had also experienced diffi culty lately in raising her right arm above her head when she was combing her hair. In a subsequent consult visit with her surgeon, she was told that a nerve was accidentally injured during the diagnostic surgical procedure and that this produced her scapular abnormality and inability to raise her arm normally. What was the origin of this nerve? ⃣ A. The upper trunk of her brachial plexus ⃣ B. The posterior division of the middle trunk ⃣ C. Roots of the brachial plexus ⃣ D. The posterior cord of the brachial plexus ⃣ E. The lateral cord of the brachial plexus
C. The long thoracic nerve was injured during the axillary dissection, resulting in paralysis of the serratus anterior. The serratus anterior is important in rotation of the scapula in raising the arm above the level of the shoulder. Its loss results in protrusion of the inferior angle (“winging” of the scapula), which is more obvious when one pushes against resistance. The long thoracic nerve arises from brachial plexus roots C5, C6, and C7. The upper trunk (C5, C6) supplies rotator and abductor muscles of the shoulder and elbow fl exors. The posterior division of the middle trunk contains C7 fi bers for distribution to extensor muscles; likewise, the posterior cord supplies extensors of the arm, forearm, and hand. The lateral cord (C5, C6, and C7) gives origin to the lateral pectoral nerve, the musculocutaneous nerve, and the lateral root of the median nerve. There is no sensory loss in the limb in this patient; injury to any of the other nerve elements listed here would be associated with specifi c dermatome losses. GAS 700-709; GA 361, 369-371
94
A 72-year-old man consulted his physician because he had noticed a thickening of the skin at the base of his left ring fi nger during the preceding 3 months. As he described it, “There appears to be some hard tissue that is pulling my little and ring fi ngers into my palm.” On examination of the palms of both hands, localized and fi rm ridges are observed in the palmar skin that extend from the middle part of the palm to the base of the ring and little fi ngers. What is the medical term for this sign? ⃣ A. Ape hand ⃣ B. Dupuytren ⃣ C. Clawhand ⃣ D. Wrist drop ⃣ E. Mallet fi nger
B. Dupuytren contracture or deformity is a result of fi bromatosis of palmar fascia, resulting in irregular thickening of the fascial attachments to the skin, which causes gradual contraction of the digits, especially digits 4 and 5. In 50% of cases, it is bilateral in occurrence. Ape hand, or fl at hand, is a result of loss of the median and ulnar nerves at the elbow, with paralysis of all long fl exors of the fi ngers and all intrinsic hand muscles. Clawhand results from paralysis of interphalangeal joint extension by interossei and lumbricals, innervated primarily by the ulnar nerve. Wrist drop occurs with radial nerve paralysis and loss of the extensors carpi radialis longus and brevis. Mallet fi nger results from detachment of the extensor mechanism from the distal phalanx of a fi nger and unopposed fl exion of that distal interphalangeal joint. GAS 758; GA 398
95
A 24-year-old female basketball player is admitted to the emergency department after an injury to her shoulder. Radiographic examination reveals a shoulder dislocation. What is the most commonly injured nerve in shoulder dislocations? ⃣ A. Axillary ⃣ B. Radial ⃣ C. Median ⃣ D. Ulnar ⃣ E. Musculocutaneous
A. The axillary nerve is a direct branch of the posterior cord and wraps around the surgical neck of the humerus to innervate the teres minor and the deltoid muscles. With this anatomic arrangement, the axillary nerve is tightly “tethered” to the proximal humerus. When the head of the humerus is dislocated, it often puts traction on the axillary nerve. GAS 674; GA 631
96
A 45-year-old male is admitted to the hospital with a painful arm after a “strongest man in the world” contest. Physical examination gives evidence of a rupture of the long tendon of the biceps brachii ( Fig. 6-5 ). Which of the following is the most likely location of the rupture? ⃣ A. Intertubercular groove ⃣ B. Midportion of the biceps muscle ⃣ C. Junction with the short head of the biceps muscle ⃣ D. Proximal end of the combined biceps muscle ⃣ E. Bony insertion of the muscle
A. The long head of the biceps brachii muscles runs in the intertubercular groove on the proximal humerus as it changes direction and turns medially to attach to the supraglenoid tubercle of the scapula. This change in direction within an osseous structure predisposes the tendon to wear and tear, particularly in people who overuse the biceps muscle. This type of injury presents with a characteristic sign called the “Popeye sign.” GAS 694, 715-716, 735; GA 370
97
After the orthopedic surgeon examined the MRI of the shoulder of a 42-year-old female he informed her that the supraspinatus muscle was injured and needed to be repaired surgically. Which of the following is true of the supraspinatus muscle? ⃣ A. It inserts on the lesser tubercle of the humerus. ⃣ B. It initiates adduction of the shoulder. ⃣ C. It is innervated chiefl y by the C5 spinal nerve. ⃣ D. It is supplied by the upper subscapular nerve. ⃣ E. It originates from the lateral border of the scapula.
C. The supraspinatus muscle inserts on the greater tubercle of the humerus and is said to initiate abduction of the arm at the shoulder. It is supplied principally by spinal nerve C5. The subscapularis muscle is the only muscle that inserts on the lesser tubercle. The subscapularis muscle is innervated by the upper and lower subscapular nerves. The teres minor takes origin from the lateral border of the scapula; the teres major takes origin from the region of the inferior angle and the lateral border of the scapula. GAS 678-679; GA 369
98
A 5-year-old boy is admitted to the emergency department after falling from a tree. The parents are informed by the radiologist that their son’s fracture is the most common fracture that occurs in children. Which of the following bones was broken? ⃣ A. Humerus ⃣ B. Radius ⃣ C. Ulna ⃣ D. Scaphoid ⃣ E. Clavicle
E. During a fall on an outstretched upper limb, the forces are conducted through the hand on up through the bones of the limb in succession. Often these bones do not fracture but rather pass the compressive forces proximally. The appendicular skeleton joins with the axial skeleton at the sternoclavicular joint. The forces are not suffi ciently transferred to the sternum, causing the clavicle to absorb the force, resulting in fracture of this sigmoidalshaped bone. GAS 673; GA 4, 6-7, 56-57, 65, 108, 351
99
A 22-year-old woman visits the outpatient clinic with pain in her left upper limb. She has a long history of pain in this limb and diffi culty with fi ne motor tasks of the hand. Physical examination reveals paraesthesia along the medial surface of the forearm and palm and weakness and atrophy of gripping muscles (long fl exors) and the intrinsic muscles of the hand. The radial pulse is diminished when her neck is rotated to the ipsilateral side (positive Adson test). What is the most likely diagnosis? ⃣ A. Erb-Duchenne paralysis ⃣ B. Aneurysm of the brachiocephalic artery, with plexus compression ⃣ C. Thoracic outlet syndrome ⃣ D. Carpal tunnel syndrome ⃣ E. Injury to the medial cord of the brachial plexus
C. The patient is suffering from thoracic outlet syndrome, involving neural and vascular elements. This results from any condition that decreases the dimensions of the superior thoracic aperture. It could be a result of a cervical rib, accessory muscles, and/or atypical connective tissue bands at the root of the neck. In this case, symptoms involve the arm, forearm, and hand. Paraesthesia along the medial forearm and hand and atrophy of long fl exors and intrinsic muscles point to a possible compression or traction problem of the lower trunk (C8, T1) rather than a lesion of either the median or ulnar nerve. The lateral palm has no sensory problem, which tends to rule out median nerve involvement. Changes in the radial pulse point to vascular compression. Erb-Duchenne paralysis of the upper trunk would affect proximal limb functions, such as arm rotation, abduction, etc. This lesion is on the left side, so the brachiocephalic artery could not be involved because it arises from the right side of the aortic arch; moreover, it would not compress the brachial plexus. Carpal tunnel syndrome would not explain the problems of the forearm and medial hand, or the long fl exor atrophy. An isolated medial cord lesion would not explain the atrophy of all long fl exors and intrinsic muscles and does not explain the radial pulse characteristics. The ischemic pain in the arm is due to vascular compression. GAS 147; GA 370
100
Physical examination reveals weakness of medial deviation of the wrist (adduction), loss of sensation on the medial side of the hand, and clawing of the fi ngers. Where is the most likely place of injury? ⃣ A. Compression of a nerve passing between the humeral and ulnar heads of origin of fl exor carpi ulnaris ⃣ B. Compression of a nerve passing at Guyon’s canal between the pisiform bone and fl exor retinaculum ⃣ C. Compression of a nerve passing through the carpal tunnel ⃣ D. Compression of a nerve passing between the ulnar and radial heads of origin of fl exor digitorum superfi cialis ⃣ E. Compression of a nerve passing deep to brachioradialis muscle
A. The ulnar nerve enters the forearm by passing between the two heads of the fl exor carpi ulnaris and descends between and innervates the fl exor carpi ulnaris (for medial wrist deviation) and fl exor digitorum profundus (medial half) muscles. Injuring the ulnar nerve results in clawhand. It enters the hand superfi cial to the fl exor retinaculum and lateral to the pisiform bone, where it is vulnerable to damage. The ulnar nerve also enters Guyon’s canal, but damage to it here would not present with the aforementioned symptoms. The median nerve enters the carpal tunnel and the radial nerve passes deep to the brachioradialis. GAS 737; GA 398
101
A 22-year-old pregnant woman was admitted emergently to the hospital after the baby had begun to appear at the introitus. The baby had presented in the breech position, and it had been necessary to exert considerable traction to complete the delivery. The newborn is shown in Fig. 6-6 . Which of the following structures was most likely injured by the trauma of childbirth? ⃣ A. Radial nerve ⃣ B. Upper trunk of the brachial plexus ⃣ C. Lower trunk of the brachial plexus ⃣ D. Median, ulnar, and radial nerves ⃣ E. Upper and lower trunks of the brachial plexus
B. During a breech delivery as described here, downward traction is applied to the shoulders and upper limbs as the baby is forcibly extracted from the birth canal. This exerts traction on the upper cord of the brachial plexus, often causing a traction injury from which the baby can often recover. If the roots of C5 and C6 are avulsed from the spinal cord, the injury is permanent. GAS 700-709; GA 361, 369-371
102
A 17-year-old female student of martial arts entered the emergency department with a complaint of pain in her hand. Patient history reveals that she had been breaking concrete blocks with her hand. Examination reveals that the patient has weak abduction and adduction of her fi ngers but has no diffi culty in fl exing them. The patient also has decreased sensation over the palmar surfaces of the fourth and fi fth digits. Which of the following best describes the nature of her injury? ⃣ A. Compression of the median nerve in the carpal tunnel ⃣ B. Fracture of the triquetrum, with injury to the dorsal ulnar nerve ⃣ C. Dislocation of a bone in the proximal row of the carpus ⃣ D. Fracture of the shaft of the fi fth metacarpal ⃣ E. Injury of the ulnar nerve in Guyon’s canal
E. Striking the concrete blocks with the medial side of her hand has injured the ulnar nerve in Guyon’s canal. This is the triangular tunnel formed by the pisiform bone medially, the fl exor retinaculum dorsally, and the deep fascia of the wrist ventrally. This injury would result in loss of sensation to the medial palm and the palmar surface of the medial one and a half digits and motor loss of the hypothenar muscles, the interossei, and the medial two lumbricals. The median nerve is not involved, for the thenar muscles and lateral palmar sensations are intact. The dorsal ulnar nerve arises proximal to the wrist, thus it would not be lost. Carpal dislocation is unlikely. If the lunate bone were dislocated, it would not cause compression of the ulnar nerve at the wrist. There is no indication of fi fth metacarpal fracture, the so-called boxer’s fracture. GAS 744; GA 417
103
A 10-year-old male suffered a dog bite that entered the common fl exor synovial sheath of his forearm. He was admitted to the hospital, where the wound was cleaned and dressed and he was treated further with rabies antiserum. Two days later the boy was suffering from an elevated temperature, and his palm and one digit were obviously swollen, causing him to cry with pain. Into which of the digits could the infection spread most easily, following the anatomy of the typical common fl exor sheath? ⃣ A. First ⃣ B. Second ⃣ C. Third ⃣ D. Fourth ⃣ E. Fifth
E. The common fl exor sheath encloses the long fl exor tendons of the fi ngers. This sheath is usually continuous with the fl exor sheath of the little fi nger, which continues within the palm, having no connection with sheaths of the other digits, which do not extend into the palm. GAS 755; GA 410
104
While sharpening his knife, a 23-year-old male soldier accidentally punctured the ventral side of the fi fth digit at the base of the distal phalanx. The wound became infected, and within a few days the infection has spread into the palm, within the sheath of the fl exor digitorum profundus tendons. If the infection were left untreated, into which of the following spaces could it most likely spread? ⃣ A. Central compartment ⃣ B. Hypothenar compartment ⃣ C. Midpalmar space ⃣ D. Thenar compartment ⃣ E. Thenar space
C. The infectious agent was introduced into the synovial sheath of the long tendons of the little (fi fth) fi nger. Proximally, this sheath runs through the midpalmar space, and infl ammatory processes typically rupture into this space unless aggressively treated with the appropriate antibiotics. GAS 768; GA 410
105
A 36-year-old patient is admitted to the emergency department with a dull ache in the shoulder or axilla ( Fig. 6-7 ). During physical examination the pain worsens by activity, and, conversely, rest and elevation relieve the pain. History reveals that the patient was hospitalized the past week and a central venous line was used. What is the most likely diagnosis? ⃣ A. Axillary-subclavian vein thrombosis ⃣ B. Compression of C5 to C8 spinal nerve ⃣ C. Disk herniation of C4 to C8 ⃣ D. Impingement syndrome ⃣ E. Injury to radial, ulnar, and median nerves
A. Axillary-subclavian vein thrombosis is becoming much more common in recent years because of the extensive use of catheters in cancer patients and other chronic medical conditions. Effort-induced thrombosis is seen with strenuous use of the dominant arm with hyperabduction and external rotation of the arm or backward and downward rotation of the shoulder as in playing cricket, volleyball, or baseball or chopping wood. Because the symptoms of subclavian stenosis are fairly dramatic, most patients present promptly, usually within 24 hours. They complain of a dull ache in the shoulder or axilla, the pain worsened by activity. Conversely, rest and elevation often relieve the pain. Patients with catheter-associated axillary-subclavian deep vein thrombosis report similar symptoms at the arm or shoulder on the side with the indwelling catheter. GAS 722; GA 377
106
A 22-year-old woman had suffered a severe knife wound to the upper lateral portion of her pectoral region, with entry of the knife at the deltopectoral groove. Pressure applied to the wound had prevented further profuse bleeding. In the emergency department, vascular clamps were applied to the axillary artery, proximal and distal to the site of injury—which had occurred between the second and third parts of the axillary artery. The vascular surgeon knew there was time to repair the wound of the artery because of the rich collateral pathway provided by the anastomoses between which of the following of arteries? ⃣ A. Transverse cervical and suprascapular ⃣ B. Posterior humeral circumfl ex and profunda brachii ⃣ C. Suprascapular and circumfl ex scapular ⃣ D. Supreme (superior) thoracic and thoracoacromial ⃣ E. Lateral thoracic and suprascapular
C. The injury is at the second part of the axillary artery. The suprascapular artery is a branch of the thy- rocervical trunk off the subclavian artery, proximal to the axillary artery. The subscapular artery is the major branch of the third part of the axillary artery, giving of the thorocodorsal and the circumfl ex scapular. In this case blood would be fl owing from the circumfl ex scapular artery in a retrograde direction into the axillary artery, supplying blood distal to the injury. GAS 695; GA 368
107
In a penetrating wound to the forearm of a 24-year-old male, the median nerve is injured at the entrance of the nerve into the forearm. Which of the following would most likely be apparent when the patient’s hand is relaxed? ⃣ A. The MCP and IP joints of the second and third digits of the hand will be in a condition of extension. ⃣ B. The third and fourth digits will be held in a slightly fl exed position. ⃣ C. The thumb will be fl exed and slightly abducted. ⃣ D. The fi rst, second, and third digits will be held in a slightly fl exed position. ⃣ E. The MCP and IP joints of the second and third digits of the hand will be in a condition of fl exion.
A. This proximal injury to the median nerve would paralyze all of the long fl exors of the digits, except for the DIP fl exors of digits 4 and 5, thereby swinging the “balance of power” to the muscles that extend the digits, all of which are innervated by the radial nerve. The intrinsic hand muscles can aid in fl exion of the MCP joints, and they are innervated by the ulnar nerve. However, they are of insuffi cient size to compensate for the extensor forces exerted on fi ngers. GAS 518, 611, 612; GA 395
108
A 55-year-old male fi refi ghter is admitted to the hospital after blunt trauma to his right axilla. Examination reveals winging of the scapula and partial paralysis of the right side of the diaphragm. Which of the following parts of the brachial plexus have been injured? ⃣ A. Cords ⃣ B. Divisions ⃣ C. Roots ⃣ D. Terminal branches ⃣ E. Trunks
C. The winged scapula results from a lesion of the long thoracic nerve, which supplies the serratus anterior muscle. This muscle is responsible for rotating the scapula upward, which occurs during abduction of the arm above the horizontal. The long thoracic nerve comes off the C5 to C7 roots of the brachial plexus. The diaphragm is supplied by the phrenic nerve, which comes off the spinal nerve roots C3 to C5. GAS 700-709; GA 361, 369-371
109
A 69-year-old man has numbness and pain in the middle three digits of his right hand at night. He retired 9 years ago after working as a carpenter for 30 years. He has atrophy of the thenar eminence (see Fig. 6-2on page 172). Which of the following conditions will be the most likely cause of this atrophy? ⃣ A. Compression of the median nerve in the carpal tunnel ⃣ B. Formation of the osteophytes that compress the ulnar nerve at the ulnar condyle ⃣ C. Hypertrophy of the triceps muscle compressing the brachial plexus ⃣ D. Osteoarthritis of the cervical spine ⃣ E. Repeated trauma to the ulnar nerve
A. The median nerve supplies sensory innervation to the thumb, index, and middle fi nger as well as to the lateral half of the ring fi nger. The median nerve also provides motor innervation to muscles of the thenar eminence. Compression of the median nerve in the carpal tunnel explains these defi cits in conjunction with normal functioning of the fl exor compartment of the forearm. The ulnar nerve is not implicated in these symptoms. Compression of the brachial plexus could not be attributed to pressure from hypertrophy of the triceps, it is located distal to the plexus. In addition, symptoms would include several upper limb defi cits rather than the focal symptoms described in this instance. Osteoarthritis of the cervical spine would also lead to increasing complexity of symptoms. GAS 764, 788; GA 406
110
A 54-year-old woman presents with pain in her right wrist that resulted when she fell forcefully on her outstretched hand. Radiographic studies indicate an anterior dislocation of a carpal bone of the proximal row (see Fig. 6-3 on page 180). Which of the following bones is most commonly dislocated? ⃣ A. Capitate ⃣ B. Lunate ⃣ C. Scaphoid ⃣ D. Pisiform ⃣ E. Triquetrum
B. The lunate bone is the most commonly dislocated bone. Displacement is always anteriorly. Dislocation of the lunate bone can precipitate the signs associated typically with carpal tunnel syndrome. GAS 752-754; GA 392, 394, 422
111
A 32-year-old male who is an expert target shooter reports pain in his right upper limb and slight tingling and numbness of all digits of the ipsilateral hand. However, the tingling and numbness of the fourth and fi fth digits is the most severe. The man states that the problem usually occurs when he is fi ring his gun with his hand overhead. Radiographic studies reveal the presence of a cervical rib and accessory scalene musculature. Which of the following structures is most likely being compressed? ⃣ A. Axillary artery ⃣ B. Upper trunk of brachial plexus ⃣ C. Subclavian artery ⃣ D. Lower trunk of brachial plexus ⃣ E. Brachiocephalic artery and lower trunk of brachial plexus
D. A cervical rib (found at C7) typically causes thoracic outlet syndrome, which is a condition characterized by weak muscle tone in the hand and loss of radial pulse when the upper limb is abducted above the shoulder. The mechanism of injury with the gun being fi red overhead suggests a lower trunk injury to the brachial plexus. The axillary artery supplies the shoulder muscles, and there is no loss of function to these muscles. The upper trunk of the brachial plexus also supplies innervation to the shoulder muscles, which are unaffected based on the patient’s presenting abnormalities. The subclavian artery is located anterior to the brachial plexus until it separates the cords as it passes under the clavicle. The brachiocephalic artery and lower trunk of the brachial plexus is only partially correct; the brachiocephalic artery is not directly associated with the brachial plexus due to its location at the midline of the body behind the sternum. GAS 700-709; GA 61, 369-371
112
A 23-year-old woman arrives at the emergency department with a swollen, painful forearm. An MRI examination reveals a compartment syndrome originating at the interosseous membrane between the radius and ulna. Which of the following type of joint will most likely be affected? ⃣ A. Synarthrosis ⃣ B. Symphysis ⃣ C. Synchondrosis ⃣ D. Trochoid ⃣ E. Ginglymus
A. A synarthrosis joint is a fi brous connection that allows minimal to no movement. In this case, virtually no movement is allowed by the interosseous membrane joint between the radius and ulna. Symphysis joints are permanent fi brocartilaginous fusions between two bones; pubic symphysis is an example. Synchondrosis is a temporary joint made of cartilage that transitions to bone typically after growth completes (i.e., epiphyseal plate). Trochoid joints are pivot joints, and the humeral-radial portion of the elbow joint is an example. Ginglymus joints are hinge joints located at the interphalangeal junctions in the hand and foot (PIPs and DIPs). GAS 731, 734-735; GA 396
113
While working out with weights, a 28-year-old woman experiences a severe pain in her chest. The pain is referred to the anterior chest wall and radiating to the mandible and her left arm. The woman felt dizzy and after 10 minutes she collapsed and was unconscious. A physician happened to be near the woman and immediately tried to feel her radial pulse. The radial artery lies between two tendons near the wrist, which are useful landmarks. Which of the following is the correct pair of tendons? ⃣ A. Flexor carpi radialis and palmaris longus ⃣ B. Flexor carpi radialis and brachioradialis ⃣ C. Brachioradialis and fl exor pollicis longus ⃣ D. Flexor pollicis longus and fl exor digitorum superfi cialis ⃣ E. Flexor pollicis longus and fl exor digitorum profundus
B. The radial pulse is best located on the anterior forearm (antebrachium) just proximal to the wrist joint. At this point the radial artery travels on the distal radius between the fl exor carpi radialis and brachioradialis tendons. The palmaris longus tendon travels more medially to the radial artery and above the fl exor retinaculum. The fl exor pollicis longus tendon is a deeper structure in the antebrachium and is also located medially to the radial artery. GAS 785; GA 374, 390
114
A 59-year-old woman is admitted to the hospital in a state of shock. During physical examination, several lacerations are noted in her forearm and her radial pulse is absent. Where is the most typical place to identify the radial artery immediately after crossing the radiocarpal joint? ⃣ A. Between the two heads of the fi rst dorsal interosseous muscles ⃣ B. At the anatomic snuffbox ⃣ C. Below the tendon of the fl exor pollicis longus ⃣ D. Between the fi rst and second interosseous muscle ⃣ E. Between the fi rst interosseous muscle and the adductor pollicis longus
B. The radial artery enters the palm through the anatomic snuffbox. The artery then moves on to pierce through the two heads of the fi rst dorsal interosseous muscle and enter the deep aspect of the palm. The fl exor pollicis longus tendon runs on the palmar aspect of the hand and the radial artery runs on the dorsal aspect of the hand before entering the UPPER LIMB 206 deep aspect of the palm, and therefore the radial artery does not run below this tendon. The radial artery does not run between the fi rst and second interosseous muscle and therefore cannot be used as a landmark to identify the artery. Finally, the artery does not run between the fi rst interosseous muscle and the adductor pollicis longus. GAS 781; GA 422
115
A 69-year-old woman visits the outpatient clinic with a complaint of numbness and tingling of her hand for the past 3 months. Physical examination reveals she has numbness and pain in the lateral three digits of her right hand that are relieved by vigorous shaking of the wrist. In addition, the abductor pollicis brevis, opponens pollicis, and the fi rst two lumbrical muscles are weakened. Sensation was decreased over the lateral palm and the volar aspect of the fi rst three digits. Which of the following nerves is most likely compressed? ⃣ A. Ulnar ⃣ B. Radial ⃣ C. Recurrent median ⃣ D. Median ⃣ E. Posterior interosseous
D. The median nerve provides innervation to the fl exor compartment of the forearm; cutaneous innervation of the second, third, and fourth digits and palmar and dorsum aspects of the hand; and innervation of four intrinsic hand muscles: fi rst and second lumbricals, abductor pollicis brevis, opponens pollicis, and fl exor pollicis brevis. The thenar compartment contains the muscle abductor pollicis brevis, opponens pollicis, and fl exor pollicis brevis, and these muscles are innervated by the recurrent branch of the median nerve. The patient has weakening of the fi rst two lumbricals and not simply the thenar muscles, so the median nerve is most likely to be compressed. Another indication that the median nerve is compressed is the vigorous shaking of the wrist. Because the median nerve traverses the carpal tunnel, carpal tunnel compression could lead to this action on part of the patient. The ulnar nerve provides innervation for part of the fl exor digitorum profundus and fl exor carpi ulnaris. These muscles are not weakened in this patient. The radial nerve provides cutaneous supply to the dorsum of the hand and forearm as well as extensor muscles of the forearm. The posterior interosseous nerve is a branch of the radial nerve and provides innervation of the extensor muscles in the forearm. GAS 724-731; GA 361
116
A 32-year-old man is admitted to the emergency department after a severe car crash. Radiographic examination reveals multiple fractures of his right upper limb. A surgical procedure is performed and metallic plates are attached to various bony fragments to restore the anatomy. Five months postoperatively the patient visits the outpatient clinic. Upon physical examination the patient can abduct his arm and extend the forearm, but the sensation of the forearm and hand is intact; however, the hand grasp is very weak, and he cannot extend his wrist against gravity. Which of the following nerves was most likely injured during the surgical procedure? ⃣ A. Posterior cord of the brachial plexus ⃣ B. Radial nerve at the distal third of the humerus ⃣ C. Radial and ulnar ⃣ D. Radial, ulnar, and median ⃣ E. Radial and musculocutaneous
B. Radial nerve at the distal third of the humerus. The patient can extend his forearm, which suggests that the triceps muscle is not weakened. Supination appears to be weak along with hand grasp and wrist drop. This would indicate that part of the radial nerve has been lost below the innervation of the triceps and above the branches to the supinator and extensors in the forearm. However, sensation on the forearm and hand is intact, indicating that the superfi cial branch of the radial nerve is intact. The superfi cial branch of the radial nerve divides from the deep radial nerve at the distal third of the humerus. The posterior cord of the brachial plexus is responsible for providing innervation of the axially and radial nerves. This patient does have some radial nerve innervation and no loss of axillary nerve function. The patient does not have weakened adduction of the wrist, indicating that the ulnar nerve is not injured. If both the radial and musculocutaneous nerves are injured, supination would not be possible as the supinator muscle and biceps provide supination of the forearm. GAS 750; GA 376
117
A 52-year-old man is admitted to the emergency department after falling on wet pavement. Radiographic examination reveals fracture of the radius. An MRI study reveals a hematoma between the fractured radius and supinator muscle. Upon physical examination the patient has weakened abduction of the thumb and extension of the metacarpophalangeal joints of the fi ngers. Which of the following nerves is most likely affected? ⃣ A. Anterior interosseous ⃣ B. Posterior interosseous ⃣ C. Radial nerve ⃣ D. Deep branch of ulnar nerve ⃣ E. Median nerve
B. The posterior interosseous nerve is an extension of the deep branch of the radial nerve. It is responsible for innervation of several muscles in the extensor compartment of the posterior aspect of the forearm, including extension of the metacarpophalangeal joints. The posterior interosseous nerve courses laterally around the radius and passes between the two heads of the supinator muscle and is thus likely to be compressed by a hematoma between the fractured radius and the supinator muscle. Though the radial nerve gives rise to the posterior interosseous nerve, this answer choice is too vague and would not indicate the precise injured branch of the radial nerve. Both the deep branch of the ulnar nerve and the median nerve traverse the medial and anteromedial aspect of the arm, respectively. These nerves primarily supply the fl exor compartment of the arm. The anterior interosseous nerve is a branch of the median nerve and supplies the fl exor digitorum profundus, fl exor pollicis longus, and the pronator quadratus. GAS 750; GA 403
118
A 34-year-old woman is admitted to the emergency department after a car crash. Radiographic studies show marked edema and hematoma of the arm, but there are no fractures. During physical examination the patient presents with inability to abduct her arm without fi rst establishing lateral momentum of the limb, and inability to fl ex the elbow and shoulder. Which of the following portions of the brachial plexus is most likely injured? ⃣ A. Superior trunk ⃣ B. Middle trunk ⃣ C. Inferior trunk ⃣ D. Lateral cord ⃣ E. Medial cord
A. The superior trunk of the brachial plexus includes C5 and C6, which give rise to the suprascapular nerve, which innervates the supraspinatus muscle. The supraspinatus muscle is the primary muscle involved in abduction of the arm from 0 ° to 15 ° . The deltoid muscle, supplied primarily by C5, abducts the arm from 15 degrees to 90 degrees. The middle trunk is just C7 and has nothing to do with the muscle involved in initial abduction of the arm. The inferior trunk is C8-T1 and does not supply the supraspinatus muscle; therefore, it is not the right answer. The cords are distal to the branching of the supraspinatus muscle; therefore, neither lateral cord nor medial cord is the correct answer. GAS 700-709; GA 361, 369-371
119
A 22-year-old man is admitted to the hospital after a car collision. Radiographic examination reveals an oblique fracture of his humerus. Upon physical examination the patient is unable to extend his forearm. The damaged nerve was most likely composed of fi bers from which of the following spinal levels? ⃣ A. C5, C6 ⃣ B. C5, C6, C7 ⃣ C. C5, C6, C7, C8, T1 ⃣ D. C6, C7, C8, T1 ⃣ E. C7, C8, T1
C. The radial nerve acts to extend the forearm at the elbow. This nerve is derived from all the roots of the brachial plexus C5 to T1. None of the other answers include all the roots and are therefore incorrect. GAS 700-709; GA 361, 369-371
120
A 56-year-old woman is admitted to the hospital after a severe car crash. A large portion of her chest wall needed to be surgically removed and replaced with a musculoosseous scapular graft involving the medial border of the scapula. Which of the following arteries will most likely recompensate the blood supply to the entire scapula? ⃣ A. Suprascapular ⃣ B. Dorsal scapular artery ⃣ C. Posterior humeral circumfl ex artery ⃣ D. Lateral thoracic ⃣ E. Supreme thoracic artery
A. The suprascapular artery arises as a major branch of the thyrocervical trunk from the subclavian artery. It has rich anastomoses with the circum- fl ex scapular artery and could provide essential blood supply to the scapula. The dorsal scapular artery would be lost with the graft. None of the other vessels listed is in position to provide adequate supply to the scapula. GAS 696, 697; GA 366, 368
121
A 56-year-old woman visits the emergency department after falling on wet pavement. Radiographic examination reveals osteoporosis and a Colles’ fracture. Which of the following carpal bones are often fractured or dislocated with a Colles’ fracture? ⃣ A. Triquetrum and scaphoid ⃣ B. Triquetrum and lunate ⃣ C. Scaphoid and lunate ⃣ D. Triquetrum, lunate, and scaphoid ⃣ E. Triquetrum and pisiform
C. The scaphoid and lunate carpal bones are in closest articulation with the radius, which is fractured in a Colles’ fracture; therefore, they would most likely be disrupted or fractured. The other carpal bones listed do not have direct contact with the radius and have a more distal location; therefore, they would not be as likely to be injured with a Colles’ fracture. GAS 734; GA 392
122
A 3-year-old girl is admitted to the emergency department with severe pain. History taking reveals that the girl was violently lifted by her raised arm by her mother to prevent the girl from walking in front of a moving car. Which of the following is most likely the cause of the pain? ⃣ A. Compression of median nerve ⃣ B. Separation of the head of radius from its articulation with trochlea of humerus ⃣ C. Separation of head of radius from its articulation with ulna and capitulum of humerus ⃣ D. Separation of ulna from its articulation with trochlea of humerus ⃣ E. Stretching of radial nerve as it passes behind medial epicondyle of humerus
C. This type of dislocation is common in children and results when the radius is dislocated and slips out from the anular ligament, which holds it in place, articulating with the ulna and the capitulum of the humerus. In adults the anular ligament has a good “grip” at the radial neck, but in young children the radial head is not fully developed, leading to an indistinct neck. Compression of the median nerve is not likely due to its medial position in the cubital fossa. The radius does not articulate with the trochlea of the humerus; the ulna articulates at this position. The ulna is not likely to be dislocated because it is more stable than the radius, which has only the anular ligament for its support. The radial nerve does not pass behind the medial epicondyle; rather, the ulnar nerve does this, so this is not the correct answer. GAS 735; GA 391
123
A 61-year-old man was hit by a cricket bat in the midhumeral region of his left arm. Physical examination reveals normal elbow motion; however, he could not extend his wrist or his metacarpophalangeal joints and he reported a loss of sensation on a small area of skin on the dorsum of the hand proximal to the fi rst two digits. Radiographic examination reveals a hairline fracture of the shaft of the humerus just distal to its midpoint. Which of the following nerves is most likely injured? ⃣ A. Median ⃣ B. Ulnar ⃣ C. Radial ⃣ D. Musculocutaneous ⃣ E. Axillary
C. Injury to the radial nerve can be caused by a blow to the midhumeral region since the nerve winds around the shaft of the humerus. The symptoms described include the loss of wrist and fi nger extension and a loss of sensation in an area of skin supplied by the radial nerve. GAS 751; GA 380
124
A 34-year-old man is admitted to the hospital after a car collision. Radiographic examination reveals a fracture at his wrist. Physical examination reveals paralysis of the muscles that act to extend the interphalangeal joints ( Fig. 6-10 ). Which of the following nerves is most likely injured? ⃣ A. Ulnar ⃣ B. Recurrent branch of median ⃣ C. Radial ⃣ D. Musculocutaneous ⃣ E. Anterior interosseous
A. The ulnar nerve innervates the dorsal and palmar interossei, which act to abduct and adduct the fi ngers and assist the lumbricals in their actions of fl exing the metacarpophalangeal joints and extending the interphalangeal joints. The recurrent branch of the median nerve innervates the thenar muscle group that functions in the movement of the thumb. The radial and musculocutaneous nerves do not innervate any muscles in the hand. The anterior interosseous innervates the fl exor pollicis longus and the pronator quadratus. GAS 770-771; GA 417
125
A 45-year-old woman is admitted to the hospital with neck pain. An MRI examination reveals a herniated disk in the cervical region. Physical examination reveals weak triceps brachii muscles. Which of the following spinal nerves is most likely injured? ⃣ A. C5 ⃣ B. C6 ⃣ C. C7 ⃣ D. C8 ⃣ E. T1
C. The triceps brachii muscle is innervated by the radial nerve (primarily C7), which comes off C5 to T1 spinal nerves. Because the patient’s only motor defi cit involves the triceps brachii muscles, one can rule out C5 and C6, which supply fi bers to the axillary, musculocutaneous, and upper subscapular nerves. Damage to either of these roots would result in additional motor defi cits of the shoulder and fl exor compartment of the arm. One can also rule out C8-T1 because these roots form the medial pectoral nerve and the medial brachial and antebrachial cutaneous nerves. Damage to these roots would result in loss of pectoral muscle function and cutaneous sensation over the medial surface of the upper limb. GAS 700-709; GA 361, 369-371
126
A 34-year-old woman is admitted to the hospital after a car collision. Physical examination reveals a mallet fi nger. Which of the following conditions is expected to be present during radiographic examination? ⃣ A. A lesion of the ulnar nerve at the distal fl exor crease of the wrist ⃣ B. A separation of the extension expansion over the middle interphalangeal joint ⃣ C. Compression of the deep ulnar nerve by dislocation of the lunate bone ⃣ D. Avulsion fracture of the dorsum of the distal phalanx ⃣ E. Fracture of the fourth or fi fth metacarpal bone
D. Mallet fi nger, also known as baseball fi nger, is a deformity in which the fi nger will be permanently fl exed at the distal interphalangeal joint, due to avulsion of the insertion of the extensor tendon at the distal phalanx. GAS 760; GA 392, 394, 422
127
A 42-year-old woman is admitted to the hospital with injury to the upper (superior) trunk of the brachial plexus. The diagnosis is Erb-Duchenne palsy. Which of the following conditions is expected to be present during physical examination? ⃣ A. Winged scapula ⃣ B. Inability to laterally rotate the arm ⃣ C. Paralysis of intrinsic muscles of the hand ⃣ D. Paraesthesia in the medial aspect of the arm ⃣ E. Loss of sensation in the dorsum of the hand
B. Injury to the superior trunk of the brachial plexus can damage nerve fi bers going to the suprascapular, axillary, and musculocutaneous nerves. Damage to the suprascapular and axillary nerves causes impaired abduction and lateral rotation of the arm. Damage to the musculocutaneous nerve causes impaired fl exion of the forearm. A winged scapula would be caused by damage to the long thoracic nerve. The long thoracic nerve is formed from spinal cord levels C5, C6, and C7, so the serratus anterior muscle would be weakened from the damage to C5 and C6, but the muscle would not be completely paralyzed. The intrinsic muscles of the hand are innervated by the ulnar nerve, which would most likely remain intact. Paraesthesia in the medial aspect of the arm would be caused by damage to the medial brachial cutaneous nerve (C8- T1; inferior trunk). Loss of sensation on the dorsum of the hand would be caused by damage to either the ulnar or radial nerves (C6 to T1). GAS 700-709; GA 361, 369-371
128
A 41-year-old woman is admitted to the hospital after a car crash. Radiographic examination reveals a transverse fracture of the radius proximal to the attachment of the pronator teres muscle. The proximal portion of the radius is deviated laterally. Which of the following muscles will most likely be responsible for this deviation? ⃣ A. Pronator teres ⃣ B. Pronator quadratus ⃣ C. Brachialis ⃣ D. Supinator ⃣ E. Brachioradialis
D. The supinator muscle attaches to the radius proximally and when fractured would cause a lateral deviation. The pronator teres originates on the medial epicondyle and coronoid process of the ulna and inserts on the middle of the lateral side of the radius, pulling the radius medially below the fracture. Pronator quadratus originates on the anterior surface of the distal ulna and inserts on the anterior surface of the distal radius, pulling the radius medially. Brachioradialis originates on the lateral supracondylar ridge of the humerus and inserts at the base of the radial styloid process, far below the fracture. Brachialis originates in the lower anterior surface of the humerus and inserts in the coronoid process and ulnar tuberosity, hence not causing an action on the radius. GAS 715; GA 379
129
A 45-year-old woman is bitten by a dog on the lateral side of her hand. Two days later the woman develops fever and swollen lymph nodes. Which of the following group of lymphatics will most likely be involved? ⃣ A. Central ⃣ B. Humeral ⃣ C. Pectoral ⃣ D. Subscapular ⃣ E. Parasternal
A. Lymph from the lateral side of the hand drains directly into humeral (epitrochlear) nodes then to the central (axillary) nodes. Pectoral nodes receive lymph mainly from the anterior thoracic wall, including most of the breast. Subscapular nodes receive lymph from the posterior aspect of the thoracic wall and scapular region. Parasternal nodes receive lymph from the lower medial quadrant of the breast. GAS 710; GA 381
130
A 25-year-old woman is admitted to the emergency department after a car collision. Radiographic examination reveals a fracture at the spiral groove of the humerus. A cast is placed, and 3 days later the patient complains of severe pain over the length of her Fig. 6-10 UPPER LIMB 189 arm. During physical examination the arm appears swollen, pale, and cool. Radial pulse is absent, and any movement of the arm causes severe pain. Which of the following conditions will most likely characterize the fi ndings of the physical examination? ⃣ A. Venous thrombosis ⃣ B. Thoracic outlet syndrome ⃣ C. Compartment syndrome ⃣ D. Raynaud’s disease ⃣ E. Injury of the radial nerve
C. Compartment syndrome is characterized by increased pressure within a confi ned space by a fascial compartment, which impairs blood supply, resulting in paleness. Venous thrombosis would not cause pain but could cause death from a pulmonary embolism. Thoracic outlet syndrome affects nerves in the brachial plexus and the subclavian artery and blood vessels between the neck and the axilla, far above the cast. Raynaud’s disease affects blood fl ow to the limbs when they are exposed to temperature changes or stress. The fracture at the radial groove probably resulted in a radial nerve injury but would not be responsible for these symptoms. GAS 744; GA 381
131
A 22-year-old woman is admitted to the hospital after falling from a tree. Radiographic examination reveals fractured pisiform and hamate bones. Which of the following nerves will most likely be injured? ⃣ A. Median ⃣ B. Recurrent median ⃣ C. Radial ⃣ D. Anterior interosseous ⃣ E. Deep ulnar
E. The deep branch of the ulnar nerve arises at the level of the pisiform bone and passes between the pisiform and the hook of the hamate, hence the ulnar is the nerve most likely to be injured in this patient. The median nerve enters the forearm between the humeral and ulnar heads of the pronator teres muscle then becomes superfi cial near the wrist. The recurrent median enters the palm through the carpal tunnel. The radial nerve divides into superfi cial and deep branches when it enters the cubital fossa. GAS 771; GA 417
132
A 43-year-old man visits the outpatient clinic with a painful shoulder. Physical examination reveals a painful arc syndrome due to supraspinatus tendinopathy. Which of the following conditions will be present during physical examination? ⃣ A. Painful abduction 0 ° to 15 ° ⃣ B. Painful abduction 0 ° to 140 ° ⃣ C. Painful abduction 70 ° to 140 ° ⃣ D. Painful abduction 15 ° to 140 ° ⃣ E. Painful abduction 40 ° to 140 °
A. The supraspinatus initiates abduction of the arm during the fi rst 15 ° of abduction; palpation of the tendon during this phase would result in pain from a tendinopathy of the supraspinatus. GAS 678; GA 361, 364
133
A 54-year-old woman is admitted to the hospital after falling from a tree with an outstretched hand. Radiographic examination reveals a wrist dislocation. Which of the following carpal bones will most likely be involved? ⃣ A. Scaphoid-lunate ⃣ B. Trapezoid-trapezium ⃣ C. Hamate-lunate ⃣ D. Pisiform-triquetrum ⃣ E. Hamate-capitate
A. The hallmark fracture caused by a fall on an outstretched hand is a scaphoid-lunate fracture; the scaphoid and lunate are the two wrist bones most proximal to the styloid process of the radius. All the other wrist bones are less likely to be affected by this injury. GAS 786; GA 392
134
A 62-year-old man is admitted to the emergency department after falling on wet pavement. Radiographic examination reveals a carpometacarpal fracture at the base of the thumb. What is the term applied to the described fracture? ⃣ A. Colles fracture ⃣ B. Scaphoid fracture ⃣ C. Bennett’s fracture ⃣ D. Smith’s fracture ⃣ E. Boxer’s fracture
C. Bennett’s fracture is a carpometacarpal fracture at the base of the thumb. Smith’s fracture is also called a reverse Colles’ fracture and is caused when the distal fragment of the radius angles forward. Colles’ fracture is also called “silver fork deformity” because the distal fragment is displaced posteriorly. Boxer’s fractures of the necks of metacarpal bones are fractures to the fi ngers. A scaphoid fracture would be indicated by pain in the anatomical snuffbox. GAS 756; GA 392
135
A 23-year-old woman is participating in a dry ski-slope competition. The woman is admitted to the emergency department after falling and catching her thumb in the matting. Radiographic and physical examinations reveal rupture of the ulnar collateral ligament of the metacarpophalangeal joint of the thumb. The thumb is extremely painful and an injection of lidocaine is performed. What is the most likely diagnosis in this case? ⃣ A. Gamekeeper’s thumb ⃣ B. Scaphoid fracture ⃣ C. Bennett’s fracture ⃣ D. Smith’s fracture ⃣ E. Boxer’s fracture
A. Interestingly, “gamekeeper’s thumb” was a term coined because this injury was most commonly associated with Scottish gamekeepers who, it is said, killed small animals such as rabbits by breaking their necks between the ground and the gamekeeper’s thumb and index fi nger. The resulting valgus force on the abducted MCP joint caused injury to the ulnar collateral ligament. These days this injury is more commonly seen in skiers who land awkwardly with their hand braced on a ski pole, causing the valgus force on the thumb as is seen in this patient. Whereas the term “skier thumb” is sometimes used, “gamekeeper’s thumb” is still in common usage. Bennett fracture is a fracture at the base of the metacarpal of the thumb. Scaphoid fracture occurs after a fall on an outstretched hand, involving the scaphoid and lunate bone. Colles’ fracture is also called silver fork deformity because the distal fragment of the radius is displaced posteriorly. Boxer’s fracture is a fracture of the necks of the second and third (and sometimes the fi fth) metacarpals. Smith’s fracture is also called a reverse Colles’ fracture and is caused when the distal radius is fractured and the distal radial fragment is angled forward. GAS 755; GA 395
136
A 54-year-old woman is found unconscious in her car. She is admitted to the hospital, and during physical examination she has absent biceps brachii refl ex. What is the spinal level of the afferent component of this refl ex? ⃣ A. C5 ⃣ B. C6 ⃣ C. C7 ⃣ D. C8 ⃣ E. T1
B. The biceps brachii refl ex involves C5 and C6 spinal nerves. C5 provides the motor component; C6 the afferent side of the refl ex arc. GAS 700-709; GA 361, 369-371
137
A 54-year-old woman is found unconscious in her bed. She is admitted to the hospital, and during physical examination she has absence of her brachioradialis refl ex. The ventral ramus of which spinal nerve is responsible for this refl ex? ⃣ A. C5 ⃣ B. C6 ⃣ C. C7 ⃣ D. C8 ⃣ E. T1
B. The brachioradialis refl ex is performed by tapping the tendon of the brachioradialis muscle. The refl ex involves spinal nerves C5, C6, and C7. The major contribution is from C6. GAS 700-709; GA 361, 369-371
138
A 55-year-old woman is admitted to the emergency department after a car crash. Physical examination reveals severe pain in the fl exor muscles of the forearm; fi xed fl exion position of the fi nger; and swelling, cyanosis, and anesthesia of the fi ngers. Which of the following is the most likely diagnosis? ⃣ A. Colles’ fracture ⃣ B. Scaphoid fracture ⃣ C. Bennett’s fracture ⃣ D. Volkmann’s ischemic contracture ⃣ E. Boxer’s fracture
D. Volkmann’s contracture is a fl exion deformity of the fi ngers and sometimes the wrist from an ischemic necrosis of the forearm fl exor muscles. Bennett’s fracture is a fracture at the base of the metacarpal of the thumb. Scaphoid fracture occurs after a fall on an outstretched hand and involves the scaphoid and lunate bones. Colles’ fracture is also called silver fork deformity because the distal fragment of the radius is displaced posteriorly. Boxer’s fracture is a fracture of the necks of the second and third (and sometimes the fi fth) metacarpals. Smith’s fracture is also called a reverse Colles’ fracture and is caused when the distal radius is fractured, with the radial fragment angled forward. GAS 762; GA 398
139
A 62-year-old man visits the outpatient clinic with pain in his hand after falling on the outstretched hand. Radiographic examination reveals a fracture of the pisiform bone and hematoma of the surrounding area. Which of the following nerves will most likely be affected? ⃣ A. Ulnar ⃣ B. Radial ⃣ C. Median ⃣ D. Deep ulnar ⃣ E. Deep radial
D. The ulnar nerve enters the forearm by passing between the two heads of the fl exor carpi ulnaris and descends between and innervates the fl exor carpi ulnaris and fl exor digitorum profundus (medial half) muscles. It enters the hand superfi cial to the fl exor retinaculum and lateral to the pisiform bone, where it is vulnerable to damage and provides the deep ulnar branch. The deep branch of the radial nerve arises proximally in the forearm. GAS 771; GA 417
140
A 32-year-old woman visits the outpatient clinic after injuring her elbow falling from her bicycle. Physical examination reveals a “benediction attitude” of the hand with the index and long fi ngers extended and the ring and little fi ngers fl exed. Which of the following is the most likely diagnosis? ⃣ A. Injury to median and radial nerves ⃣ B. Injury to median nerve ⃣ C. Injury to radial and ulnar nerves ⃣ D. Injury to ulnar nerve ⃣ E. Injury to median ulnar and radial nerves
B. “Benediction attitude” of the hand with the index and long fi ngers straight and the ring and little fi ngers fl exed is caused by an injury to the median nerve. The long fl exors of the digits are supplied by the median nerve; the unopposed radial nerve and deep ulnar nerve supply the extensors of the digits 1–3, causing them to be in the extended position. Digits 4 and 5 are slightly fl exed, because the fl exors of the PIP joints are supplied by the ulnar nerve. GAS 724; GA 371, 376
141
A 54-year-old man is admitted to the emergency department with severe chest pain. Electrocardiographic evaluation reveals a myocardial infarction. Due to the severity of the infarction, a coronary artery bypass surgery using a radial artery graft is proposed. Which of the following tests should be performed during physical examination prior to the bypass graft operation? ⃣ A. Allen test ⃣ B. Triceps reflex ⃣ C. Tinel test ⃣ D. Brachioradialis reflex ⃣ E. Biceps reflex
A. The Allen test involves compression of the radial and ulnar arteries at the wrist with the fi ngers fl exed tightly. Pressure is then released on either vessel successively to determine the degree of supply to the hand by either vessel and the patency of the anastomoses between them. The usefulness of the radial artery for bypass can thereby be assessed. The other tests have nothing to do with the patency of the radial artery. GAS 770; GA 400
142
A 34-year-old man visits the outpatient clinic with a painful upper limb after a fall onto a concrete fl oor. Physical examination reveals that the patient has weak abduction and adduction of his fi ngers but has no diffi culty in fl exing them. The patient also has decreased sensation over the palmar surface of the fourth and fi fth fi ngers. Which of the following diagnoses is most likely? ⃣ A. Compression of the median nerve in the carpal tunnel ⃣ B. Injury of the radial nerve from fractured humerus in the radial tuberosity ⃣ C. Compression of the median nerve as it passes between the two heads of the pronator teres ⃣ D. Compression of the radial nerve from the supinator ⃣ E. Injury of the ulnar nerve by a fractured pisiform
E. The ulnar nerve enters the hand superfi cial to the fl exor retinaculum and lateral to the pisiform bone and innervates all the interossei via the deep branch. These muscles are responsible for adduction and abduction of the fi ngers. Flexion of the fi ngers is spared because the fl exor digitorum superfi cialis and most of the fl exor digitorum profundus are innervated by the median nerve, which is unaffected by this injury. Had the median nerve been compressed in the carpal tunnel, one would have diffi culty with motion of the thumb as a result of a lack of innervation of the thenar muscles. An injury of the radial nerve in the arm will result in extension defi cit in the forearm and hand. GAS 744; GA 417